You are on page 1of 139

Philippine nursing licensure

Examination (pnle)

Reviewer

Compilation of board exam


November 21 & 22, 2021

Subjects covered

Nursing practice i: community health nursing

Nursing practice ii: care of healthy/at risk mother and child

Nursing practice iii: care of clients with physiologic and psychosocial


alterations, part a

Nursing practice iv: care of clients with physiologic and psychosocial alterations,
part b

Nursing practice v: care of clients with physiologic and psychosocial alterations,


part c

Compiled by: ambagan


Nursing practice i: community health nursing

Situation – nurse nona is a newly hired community health nurse. At the start
of her duty, she takes into consideration that she should assume numerous
roles and functions that are primarily focused on promoting a healthy
community.

1. In order for nurse nona to become effective in her caregiving role to


the different type of clientele in the community, she should be equipped
with the basic knowledge about _______.
a. Statistics
b. Research
c. Nursing process
d. Nursing theories

2. As planner and programmer in the community setting, nurse nona should


understand that the primary objective of community health nursing is
focused on the importance of _______.
a. Curative aspect of care
b. Restoration of full function
c. Rehabilitative part of care
d. Optimum health and wellness

3. The best community health nursing principle that will guide nurse nona
in the effective performance of her various roles and function is
focused on the community that _____.
a. Has different health needs and problems
b. Is considered as a primary client
c. Has various structures and resources
d. Is composed of individudals and families

4. As health monitor, which of the following should be considered by the nurse


as health threats in the community?

1. Ineffective breastfeeding
2. Fire hazards
3. Inadequate immunization
4. Polluted water supply

a. 3,4
b. 2,3,4
c. 1,2,3,4
d. 1,2

5. Which is the initial task of nurse nona as a community organizer?


a. Formulates individuals, family groups and community care plan
b. Motivates and enhances community participation
c. Prepares and submits required reports on time
d. Coordinates with individuals, families, groups for health services

Situation – nurse bella takes charge of handling the mental health promotion
program in the midst of the ongoing health crisis due to the covid 19
pandemic.

6. Teaching the families on promoting mental health at home in this


pandemic time should focus on the following except:
a. Seeking community support
b. Observing open communication
c. Engaging social media the whole day
d. Reaching out to friends

7. During this pandemic time, which of the following roles of nurse bella
should she significantly intensify to prevent more cases of mental
problems in the community?
a. Case finder
b. Case manager
c. Researcher
d. Surveyor

8. There are individudals who are suffering from mental health challenges
due to the pandemic. The best nursing action is _____.
a. Refer for admission at the mental health unit
b. Help in setting up debriefing stations
c. Recommend hiring of a community psychiatrist
d. Set up a mental health program

9. Which of the following information about severity of mental illness


would be important to the mental health program in the community
setting?
a. Mentally-ill patients are always dangerous
b. Mental illness is hereditary
c. Mental illness is not curable
d. Mental illness is a global concern

10. After a webinar on the mental health act, nurse bella realized that the
community has to change which of the following perceptions?
a. Mental illness is not hereditary
b. Community resilience is important
c. Mental illness is incurable
d. Psychosocial services should be available

Situation - nurse rona recognizes the importance of the application of


epidemiology in community health setting.

11. In priority in managing community health needs is through knowing in


depth the prevalence of diseases because it indicates the _____.
a. Risks to health problems
b. Presence of health problems
c. Priority needs of the people
d. Magnitude of health problems

12. nurse rona and her team has been utilizing the “epidemiologic triad”
model of identifying causative factors of diseases. Which of the
following is not relevant to this model?
a. External agent
b. Susceptible host
c. Treatment regimen
d. Environment

13. there is an outbreak of measles in some areas of the community where


investigation should nurse rona and her team begin?
a. Identify and count cases
b. Verify diagnosis
c. Define and identify diagnosis
d. Prepare for field work

14. nurse rona is able to know the patterns of occurrence and distribution
of diseases in the community. Per record she found out there had been
cases of rabies at irregular intervals scattered in particular places.
This occurrence refers to _____.
a. Endemic
b. Epidemic
c. Pandemic
d. Sporadic

15. as an epidemiology nurse, nurse rona’s primary function and


responsibility is to _____.
a. Assist the epidemiologist in making reports
b. Implement public health surveillance
c. Render nursing care to sick residents
d. Follow up cases and contacts

Situation – nurse danica is assigned to the health education initiative of


the health center.

16. nurse danica would like to put more emphasis on healthy lifestyle in
Her health education classes. Which step in the teaching-learning process
should she first start?
a. Assessing health needs
b. Evaluation of learning takes place
c. Preparing learning materials
d. Developing health education plan

17. in planning for the health education program about healthy lifestyle
Nurse danica targets to work with parents in the community. Noting different
ethnics backgrounds among them, which important consideration should be part
of the plan?
a. Economic
b. Social
c. Cultural
d. Educational

18. based on the theory of adult learning by knowle’s, which of the


following principles will guide the nurse to be an effective educator?
a. Educational attainment
b. School where the participants graduated
c. Age and gender of learners
d. Environment and learner’s participation

19. Nurse danica plans to have the residents of the community conduct
brainstorming about their community problems. Which of the following is
the rationale of this teaching format?
a. Encourages problem solving and critical thinking skills
b. Links audience participation to the presenter’s speaking style and
content
c. Allows the participants to generate ideas and discuss them in group
setting
d. Offers shared experiences in a familiar setting

20. Successful teaching and learning activities in any health care setting
is primarily fostered through _____.
a. Identifying motivating factors
b. Using clear and concise language
c. Establishing trust and rapport
d. Setting realistic goals and objectives

Situation – nurse keena is alarmed with the rising number of re-emerging


cases of pulmonary tuberculosis (ptb) in the philippines.

21. Case finding for ptb in the community requires that the nurse should
identify persons having sputum characterized as _______.
A. Rusty, frothy
B. Yellowish
C. Blood stained
D. Greenish

22. In the care of patients with communication diseases, nurse keena should
know that the feces, urine, blood and other body fluids are considered as
A. Portal of entry
B. Vehicles of transmission
C. Reservoir of agent
D. Portal of exit

23. With the onset of the rainy season, the nurses are closely monitoring the
increase of dengue cases. They should focus their health teaching on ways to
prevent dengue by _____.
A. Destroying breeding places of the vector
B. Using repellant lotion
C. Burning of dried leaves
D. Wearing of protective clothing
24. Outbreak of cases of typhoid fever occurs in the community. Nurse keena
should inform the residents that the transmission of the disease is through
______:
A. A vector
B. Food and water
C. Blood and body fluids
D. Air

25. There are several cases of rabbies in the community. Which of the
following preventive measure should nurse keena teach the families?
A. Impound all dogs
B. Impose “no pets allowed” rule
C. Have the dogs vaccinated
D. Report to the authorities households with dog/s

Situation – as community health nurse it is expected that she is competent in


performing core competency standards on record management.

26. Nurse ana is aware that the domain of record management entails which of
the following except:
A. Write in the kardex what was to be endorsed
B. Keep human resources update at all times
C. Report to md orally wrong use of drug
D. Ensure entries in patient record is signed

27. Improvement of nursing service as well as patient satisfaction is also a


concern of the nurse manager. Which of the following strategies can be adopted
to achieve this goal
A. Hold regular meeting with staff
B. Make chart audit
C. Plan suggestion box
D. Conduct regular rounds of patients

28. A physician in the health facility has forgotten to change antibiotics of


the patient and make phone call order to nurse ana. What should be the
appropriate action of the nurse in this situation?
A. Ensure that the order will besigned by the physician as soon as heis nach
to the facility.
B. Have any physician in the facilyt sign the order
C. Carry all phone orders and bring chart to the doctor for signing
D. Call back md to give feedback

29. If the chn has written in the treatment record : bp 160/96, ace inhibitor
with diazide admistered, bp decrease at 140/88 and urine output 1200 cc for
whole shift. What type of charting is this?
A. Pie
B. Aira
C. Soap
D. Dar

30. Which of the following is primary purpose of accurate, updated and


reliable document?

A. Use from the shift report of nurse and doctor

B. Protect nurse from doctors’ reprimand

C. Serve as evidence of legal matters

D. Audit frequency of medication utilization

31. Which of the following is the rationale why team work is essential in
health care

A. Helping one another

B. Makes work lighter

C. Patient safety

D. Promotes camaraderie

32. What are the main component of collaboration?


A. Shared vision
B. Achievement of goals
C. Working together
D. Partnership

33. Nurse roy as member of the health team describes himself as good team
player, which of the following is the best characteristics of good team
player?
1. Sence of humor
2. Good listener
3. Inspires others
4. Proactive thinking
A. 1,2,3,4
B. 1,2,3
C. 1,2
D. 3,4

34. Which of the following is an essential element of effective collaboation?


A. Communication
B. Belong to one team
C. Networking
D. Delegation

35. Nurse roy in his dealings with his co-workers instills collaborative
partnership in doing their task which is the aim of this type of partnership?

A. Make work faster and easier


B. Shared goals
C. Change and influence group behavior
D. Staffs’ satisfaction

Situation – nurse nancy is in charge of the geriatric care program of the


barangay.

36. To plan out for the activities intended for the elderly population the
nurse should begin
to______

A. Do survey of the number of the elderly population


B. Get informed consent
C. Ask permission from barangay officials
D. Tap the help of expert

37. In assessing the health condition of the older persons, which is the best
measure in
determining their functional status?

A. Financial capability of the family


B. Age of the patient
C. Activities of daily living
D. Educational attainment

38. Which information should the nurse provide to the elderly and their
families regarding drug regimen presribed to patient?

A. Liquid medication is more convenient and safer


B. Allow them to refuse to take medicines
C. Give independence to take medicines
D. They are more vulnerable to adverse drug reaction

39. care giving to older persons can be burdensome to other members of


family who are the caregivers. During home visit to the family, the nurse
should look for symptoms of ____.
A. Agitation
B. Burnout
C. Withdrawal
D. Suicidal tendency

40. For an effective program for the elderly population, which should be the
appropriate approach?

A. Assign to the family


B. Supervised by physician
C. Multidisciplinary
D. Under the care of barangay health workers

Situation – nurse zoe as a public health nurse together with other members of
the health team ensure there is quality in health and nursing care, in their
work place.

41. Which of the following is an important tool of assessing quality in the


delivery of health services?
A. Research
B. Standards
C. Nursing process
D. Legislation
42. Choose the key words that best describes quality improvement in public
health.
A. Status quo
B. Imporvement process
C. Very conventional
D. Perfect health facility

43. Choose the description that quality exists at the health center
A. Patient satisfaction
B. Selected delivery of services
C. Controlled by local officials
D. Rapid turnover

44. Nurse elisa would like to enhance the process standard at the health
center. Which should be her priority nursing intiative ?
A. Revise manual for operation
B. Hire more new graduate
C. Upgrade the health equipment
D. Reduce operational expenses

45. Identify to whom nurse elisa would seek help to improve health resources
like mdicines in orded to provide quality nursing care at the health center.
A. Head of the church
B. Secretary of health
C. Mayor
D. Governor

Situation – nurse elisa has been promoted as nurse manager in the health
unit. Her functions include management of resources and environment and
budgeting.

46. As nurse manager, which should she consider important when preparing the
budget needed for the operation of the unit.
A. All things and equipment need to be new yearly
B. Budget is implemented with or without money at hand
C. Cost effectiveness and efficiency are important
D. Budget is not essential for quality assurance

47. In the process of planning, which of the following, nurse elisa should use
the plan that would review the strengths and wekness of the organization
A. Operational
B. Nursing care
C. Strategic
D. Program

48. For cost effectiveness measure, which of the following is the best nursing
action
A. All things should be kept at the supervisors office
B. Identify resources needed to accomplish task
C. Assign the utility worker to keep track
D. Allow everyone to take responsibility

49. Nurse elisa advocates for safe staffing pattern at the health center.
Which best practice she should instill from the staff?
A. Delegate task
B. Give all freedom to be honest in their attendance
C. Staffing pattern not important in community setting
D. Requires staff to report on time

50. Which of the following is a measure of safe work environment?


1. Complies with standars
2. Observe protocols
3. Adheres to policies
4. Observe quality
A. 2,3,4
B. 1,2
C. 3,4
D. 1,2,3,4

Situation – when prioritizing problem in the community, the problems are


categorized as health status, health resources or health related.
51. Nurse maris is correct in identifying, which of the following is a health
resource problem ?
A. Feud between the midwife and head of the barangay sanitation committee
B. High maternal mortality rate
C. Absence of midwife in the community to render health services
D. Increase in number of deaths from pneumonia

52. There are five criteria in prioritizing community health problems. If


bnurse maris is estimating the proportion of the population affected by the
problem, she is using what criterion in prioritization?
A. Magnitude of the problem
B. Social concern
C. Nature of the problem
D. Modifiability of the problem

53. Select a barrier to goal setting between the nurse and the family.
A. Educational attainment
B. Socio economic status
C. Nature of employment
D. Failure of family to perceive existence of problem

54. Which is the most appropriate intervention should the nurse do to help
family perform the health tasks?
A. Refer family to barangay officials for guidance
B. Help the family recognize the problem
C. Allow family to decide to use health resources
D. Leave the family what action take on their problem

55. Choose the step of nursing process that identifies the family health
seeking behavior
A. Implementation
B. Evaluation
C. Assessment
D. Planning
Situation – nurse oscar is in charge of the osteoporosis control program of
the health center

56. Nurse oscar inform the other members of the team about osteoporosis. Which
of the following is true of the disease in the philippines
A. There are only few cases in the country
B. The disease is not alarming
C. Filipinos are aware of the incidence of osteporosis
D. Filipinos have low awareness about the disease

57. Which is the most important risk factor to osteoporosis


A. Menopause
B. Being male
C. With history of previous fracture
D. Short in height

58. Which of the following is the preventive measure to osteoporosis


development
A. Iron rich food
B. Calcium rich food and supplement
C. Daily jogging
D. Vigorious exercise

59. Which of the following is true about osteoporosis


A. It is a silent disease
B. It is more common in men
C. It is not htat alarming
D. It is only discovered after a fracture

60. The diagnostic examination for osteoporosis is geared towards measuring


A. Bone density
B. Blood volume
C. Weight
D. Height
Situation - nurse telly is planning to conduct a qualitative research about
the life experiences of teenage mothers in the community.

61. Which “appropriate” research design nurse telly should use in her
planned study?

A. Grounded theory

B.phenomenology

C.case study

D.etnography

62. After shed has decided to conduct the study, which of the following
nursing action she should do next?

A. Create conceptual framework

B.make a research proposal

C.decide for the data analysis

D.plan for data collection

63. Which is the “appropriate” means to present qualitative data?

A.themes

B.bar graph

C.pie graph

D.linear graph
64. Which of the following nursing actions nurse telly must do in doing data
saturation of qualitative data

A.do resurvey

B.use case study

C.do interview more than once

D.seek advice of expert

65. One of the study questions in the proposed study is about needs of
teenage mothers.in choosing framework to make the topice more stable,which
theory nurse telly should adopt in her study?

A.roy

B.maslows

C.orem

D.kings

Situation - nurse rina had been working as public health nurse,in working with
families she valules the importance of family care plan, and other concepts
about community health nursing

66. Nurse rica is in charge of the animal bite program fo the health
center.which of the following is the causative agent of rabies?

A.parasite

B.virus

C.bacteria

D.fungi
67. Which is the best strategy to control incidence of rabies in the
community?

A.impound dogs

B.kill stray dogs

C.responsible pet owner

D.fine the owners

68. Which best advice the nurse should give in case of dogbite?

A.wash wound with garlic

B.do not feed the dog

C.bleed the bitten area

D.wash thoroughly with running water

69. What advice should be given to the owner of the dog in case of dogbite

A.give away animal

B.impound the dog

C.kill the dog

D.feed well and observe

70. Which important information the nurse should inform the public about
rabies?

A.it could be prevented

B.it is an ordinary disease


C.rabies is not deadly

D.it kills

Situation - nurse erica is fully aware that being a public health nurse her
work is guided by ethico-moral principles.

71. Which is the 'main' goal of ethical practice of the nrusing profession
including the community setting?

A.to protect the nurse and co workers

B.to prevent reprimand from physicians

C.for the patients family satisfaction

D.centered on the welfare of clients and protect their rights

72. As public health nurse nurse erica makes sure all the supplies and
medicines needed for the care of community are available. This is an example
of the principle of ______________.

A.justice

B.respect

C.fair treatment

D.benificence

73. Select nurses action in keeping with principle of confidentiality

A.hides identitt of patient

B.shares information from patients chart in public


C.keeps all matters about the patient as a secret

D.discusses the case of patient with others

74. Nurse erica is always guided by the principle of benificence in all that
she does to all her patients. Which of the following nursing action is aligned
with principle of beneficence.

A. Equal and fair allocation of resources to all

B.getting informed consent

C.keep records of patients from public viewing

D.promoting patients safety at all times

75. Nurse erica tap all the newly hired nurses to be members of her new
prohect. She is observing the ethical principle of __________.

A. Justice

B.autonomy

C.nonmaleficence

D.respect

Situation – nurse henie uses the family coping index in assessing family
health needs and problems of particularly of families who are vulnerable to
illness.

76. Which of the following is an evidence of a poor family coping index


related to healthcare attitudes?

A.a parent who washes the wound of the child with running water.
B.the family who observes the habit of cleaning surroundings.

C.a young mother whi introduced solid food to her three-month old baby.

D.a mother who brings her child to be vaccinated for measles.

77. Which of the follwing is the best evidence of a family whose family
coping index on therapeutic competence is rated as coping well?

A.shows positive interpersonal relationshiop

B.participates in the weekly clean and green program of the community

C.maintain clean and organized household ambiance

D.visits the clinic frequently well or sick.

78. Which of the following tools used by nurses in the community setting for
assessing health needs and problems of families that is similar to family
coping index

A.nursing theories

B.vital statistics

C.case study

D.nursing diagnosis

79. In utilizing family coping index nurses should be knowledgeable that the
focus of this tool is identifying the family's _____________.

A.strategies to deal with stressful situations

B.manner of interaction

C.ways of managing health needs and challenges

D.patterns of health habits


80. Nurse myrna is taking care of a firmly whose there young children are
sick with malnutrition particularly protein deficiency. Which of the following
behaviors is indicative of the family's positive coping index

A.observing erratic mealtime due to work schedule of parents.

B.bringing children to health center for consult only when needed.

C.serving foods that children like but high in carbohydrates.

D.cooking foods in variety that include meat,dairy products and beans.

Situation – ms. Sandra is a newly assigned school nurse in the elementary


school of the barangay. She planned to do physical examination of every child.

81. In assessing the health condition of school children, which of the


following would be the findings common to this age group?

A. Cancer and other malignancies

B. Anemia and other blood disorders


C. Lice and parasites
D. Fracture and injuries

82. In the performance of her roles and functions as a school nurse, which
guiding principle should she consider very important?

A. Health is an integral part of the education process.


B. The nurse is in full authority over the children.
C. The local health authority supervises the school health.
D. School nursing is more focused on socialization.

83. Choose how many times at least the nurse conducts physical assessment to
school children.
A. Thrice a year
B. Every semester
C. Every quarter
D. Once a year

84. Who should nurse sandra consider as a priority for home visitation?

A. Pedro 9 years old whose parents are both working.


B. Melissa 10 years old with stunted growth for her age.
C. Cindy 7 years old who has been absent due to skin lesions.
D. Mike 8 years old who often sleeps during class hour.

85. Which of the indicators best describes an effective outcome of school


nursing programs and initiatives.

A. Zero absenteeism and tardiness of pupils.


B. Teachers are observers of school health program.
C. Limited information in school of health initiatives.
D. Constant and visits and phone calls of parents.

Situation - nurse cris is about to start her community organizing activities.


Together with her core group, their goal is to be successful in community
organization through effective communication.

86. Nurse cris would like to communicate to the people the health problems
they had identified. In reaching out every household in the community which
is the best strategy the nurse should employ?

A. Call for general assembly


B. Use social media
C. Send memorandum
D. Write a letter to residents

87. Which communication techniques should nurse cris employ in order to


successfully capture the details of the meeting?

A. Summarizing

B. Restating
C. Reassuring

D. Validating

88. Nurse cris received information from some community residents who suspect
that a neighbor is abusing his young child. Which should be the priority
nursing action?

A. Report to police authorities.


B. Notify the social worker.
C. Ignore the information.
D. Validate the information

89. In organizing the community in the midst of a major health crisis, which
strategy should the nurse cris employ to effectively communicate the
importance of health and safety protocol?

A. Send letter compelling everyone to follow.


B. Use social media to spread the information.
C. Request an elder residents to inform everyone.
D. Place poster and flyer in dialect in strategic places.

90. In order for nurse cris to facilitate the recognition of the community of
the existence of their health problems, which nursing action would yield
better results?

A. Asks the barangay head to make the report.

B. Set the ground rule that presence of problem is valid

C. Allows people’s participation to confirm the health.

D. Acts as an expert to communicate to residents.

Situation – nurse bea engages in both formal and informal learning activities
to enhance her knowledge about nursing theories and other concept in
nursing.
91. Nurse bea recall the theory of nursing as caring by _____________.

A. Orem
B. Watson
C. Kings
D. Banners

92. Which of the following is the central theme of sr. Calista roys theory.

A. Self-care deficit

B. Adaptation

C. Nursing as caring

D. Transcultural

93. This theory categorizes professionals as novice to expert is by


____________.

A. Benner

B. Abdellah

C. Pender

D. Kings

94. Nurse bea reviewed the elements of nursing as a profession. Which of the
following is an important element that characterizes nursing as a
profession?

A. Has members
B. Possess body of knowledge
C. Service oriented
D. A calling
95. Nurse bea wants to pursue higher education in nursing for her career
advancement. Aside from enrolling in graduate school, which of the
following she could enhance her career?

A. Participates in professional organization


B. Attends seminars
C. Conducts research
D. Attend symposium

Situation - nurse levy reviews all pertinent laws that affects public health
nursing.

96. Ra 9173 is otherwise known as the philippine nursing act of 2002. Which is
the primary aim of this law?

A. To enhance the competence of professional nurses.


B. To regulate practice of professional nursing in the country
C. To facilitate mobility of nurses to other countries
D. To promote well – being of health workers.

97. The nurse closely monitors the work she delegated to the barangay health
worker. She is legally guided by his principle

A. Respondent superior
B. The good samaritan
C. Res ipsa loquitor
D. Jurisprudence

98. This law promotes the wellbeing and living conditions of health workers
especially those from the government managed facilities ________.

A. Continuing professional development


B. Magna carta for health workers
C. Philippine qualifications framework
D. Local government code

99. It mandates the compulsory immunization of children below 8 years

A. Pd 996
B. Ra 11223
C. Ra 8173
D. Ra 10912

100. This law allows every filipino to avail of affordable medicines.

A. Local government code e


B. Primary health care
C. Universal health car
D. Generic drug act
Nursing practice ii: care of healthy/at risk mother and child

Situation - nurse kathy is caring for a postpartum patient. Routine postpartum


care is rendered to the patient.

1. Which assessnebt finding would lead the nurse to suspect a postpartum


hemorrhage? Blood loss of __

A. Less than 300 ml/24 hours


B. More than 400 ml/24 hours
C. Less than 200 ml/24 hours
D. More than 500 ml/24 hours

2. Which of the following is caused by the markedly distended uterus and


intermittent uterine contractions within 2 to 3 days after birth?
A. Retained placenta
B. Uterine atony
C. After pains
D. Boggy uterus

3. The nurse prepares a care plan for the patient. Based on ramona mercer’s
becoming a mother (bam) theory, which of the following statements foster the
process of becoming a mother?
A. The woman becomes comfortable with her identity as a married individual.
B. It encompasses the dynamic transformation and evolution of a woman’s
persona.
C. A woman learns mothering behavior prior as early as a teenager.
D. It accurately reflects the transitional process from being single to a
married relationship.
4. The mother asks why she has a gush of blood coming out from the vagina that
occurs when she first arises from bad. The nurse’s correct response should be
_______:
A. “blood pools at the top of the vagina and forms clots that are passed upon
rising or sitting on the toilet.”
B. “positioning causes blood to flow out when she stand.”
C. “because of the normal pooling of blood in the vagina when the woman lies
down to rest or sleep.”
D. “normal physiologic occurrence that results as the body attempts to
climinate excess fluids.”

5. Some postpartum mothers will experience difficulty voiding because of the


edema and trauma of the perineum. Which priority nursing measures stimulate
the sensation of voinding?
A. Encouraging her to avoid.
B. Running water in the sink or shower.
C. Helping the mother into the shower.
D. Providing cold tea or fluids of choice.

Situation - a postpartum mother newly delivered her baby per vagina. She
keeps on asking the nurse when the basic physiologic changes occur as her body
returns to a prepregnant state.

6. The nurse explains to the mother that the uterus will return to its
prepregnancy state in ___ weeks.
A. Six
B. Three
C. Four
D. Five

7. In her capacity to teach, the nurse describes the changes of the uterus
after childbirth to return to a nonpregnant state as ____.
A. Catabolism
B. Subinvolution
C. Contraction of muscle fibers
D. Involution

8. Which of the following conditions does the nurse explain to the patient the
contributory factor that slows uterine involution?
A. Full bladder during labor
B. Difficult birth
C. Prolonged labor
D. Infection during pregnancy

9. The nurse assesses the uterine funds of the mother. Which part of the
abdomen will nurse begin?
A. Symphysis pubis
B. Midline
C. Umbilicus
D. Sides of the abdomen

10. The first priority nursing intervention during the immediate postpartum
period is focused on ___.
A. Monitoring urinary output
B. Taking the vital signs every 4 hours
C. Observing postpartum hemorrhage
D. Checking level of responsiveness

Situation - evelyn a multigravida, in her 20th weeks of generation visited the


community clinic with complains of dizziness, vertigo, and heartburn. After
the physical assessment, nurse harper finds the patient as malnourished.

11. Iron supplementation was prescribed because of her low hemoglobin level.
Which statement, if made by evelyn, would indicate an understanding of health
instruction?
A. “my body has all the iron it needs and i don’t need to take
supplements.”
B. “meat does not provide iron and should be avoided.”
C. “the iron is best absorbed if taken on an empty stomach.”
D. “iron supplements will give green color to my stool.”

12. Evelyn was given iron as supplemental vitamin to prevent maternal anemia.
She asks if it will not be affected because she is regularly taking vitamins
c. Which of the following would be the best response of the nurse?
A. “take two other vitamins separately.”
B. “take the iron after a full meal.”
C.”Absorption of iron is enhanced with vit c.
D. “drink milk when taking the iron supplement.”
13. Evelyn was advised to take calcium supplements on the 2nd and 3rd trimester
of pregnancy. Which of the following would enchance her intestinal absorption
of calcium?
A. Fat-soluble vitamins
B. Proteins
C. Minerals
D. Water soluble vitamins

14. Nurse harper observed evelyn has knowledge deficit regarding fetal
nutrition. Nurse narper has to explain that the main source of nutrition for
the baby is which of the following?
A. Amniotic fluid
B. Uterus
C. Placenta
D. Chorionic villi

15. Nurse harper provides health instruction to the patient experiencing


heartburn. Which statement by the patient indicates a need for further
instruction? I have to _____.
A. Drink milk between meals
B. Eat small, frequent meals
C. Avoid fatty or spicy foods
D. Lie down after eating

Situation - the giving of medication to a pediatric patient is a serious


responsibility of a nurse. Nurse imelda has just been assigned to the
pediatric wards.

16. When giving medicine to pediatric patients, dosage varies. Which of the
following should nurse imelda consider?
A. Height and surface area
B. Size, surface area & age
C. Size, surface area, age & height
D. Size & surface area

17. The headnurse checks nurse imelda’s knowledge on administering oral


medication to pediatric patients. Which of the following statements below
should she choose as correct?
A. A child’s reaction to a dose ordered by a physician is not less
predictable than adult’s reaction.
B. When giving oral medication, the child as young as two years of age cannot
be taught to swallow drugs.
C. The child should be told to plate the tablet in the middle of his tongue
and drink water to wash down the tablet.
D. The possibility of error is greater in the giving of medication to children
than to adults

18. In infants and toddlers, which part should nurse imelda often use for
intramuscular injection to reduce the risk of vascular and peripheral nerve
injuries?
A. Gluteus maximus
B. Dorso-gluteal
C. Deltoid muscle
D. Vastus lateralis

19. Administering medication intramuscularly can produce a variety of serious


adverse effects has been revealed in comprehensive surveys of research
reports. When asked by the headnurse what is the most common complication that
may arise, nurse imelda should mention ____.
A. Abscess
B. Herve palsies and paralysis
C. Hematoma
D. Muscle contracture

20. Prior to administering the drugs ordered by the pediatrician, nurse imelda
needs to know if she is giving the ordered medication to the right patient.
The first step is __________.
A. Check the patient’s hospital bracelet
B. Ask the parent/significant other to state name of patient and birth date of
patient.
C. Verify patient’s allergies with chart and with patient
D. Compare medication order to identification bracelet.

Situation – alaia, a patient with nevere proeclampsia, is admitted to the


hospital. She is a student from one of the local universities, she insists of
continuing her studies while in the hospital despite being instructed to rest.
The patient studies approximately 10 hours a day and has numerous visits from
fellow students, family, and friends.
21. Nurse isabelle is concerned about the patient’s welfare and her ability
to comply with the doctor’s instructions. What should be the appropriate
action?

A. Include a significant other in helping the patient


Understand the need for rest.
B. Instruct the patient that the baby’s health is more
Important than her studies at this time.
C. Develop a routine with the patient to balance her studies
And her rest needs.
D. Ask her why she is not complying with the prescription for bed rest.

22. Patient alaia, who seems to be irritated with the nurse, said, “i don’t
want to talk to you because you’re only a nurse. I will wait for my doctor.”
Which of the following is an appropriate response by the nurse?

A. “i’m angry with the way you dismiss me.”


B. “so then you would prefer to speak with your doctor?”
C. “i understand. I should call your doctor.”
D. “your doctor prescribed this for us to do nursing care.”

23. Nurse alaia is now in a dilemma. This occurs when .

A. There is a conflict between the nurse’s decision and that of his/her


superior193
B. Choices are unclear
C. There is a conflict of two or more ethical principles
D. A decision had to be made quickly under a stressful
Situation

24. Which of the ethical principles stipulates that the nurse is responsible
for providing all patients with are, attention and information?
A. Beneficence c. Nonmaleficence
B. Advocacy d. Veracity

25. Which action by the nurse provides a safe environment for a preeclamptic
patient?

A. Maintain fluid and sodium restrictions.


B. Take off the room lights and draw the window shades.
C. Encourage visits from family and friends for psychological
Support.
D. Take the patient’s vital signs every four hours.

Situation – part ii of the training is the giving of the hypothetical


situation for the application of what was taken during the didactic. A group
was given a scenario of a pregnant woman in the ob ward.

26. The scenario states that the nurse is discussing the nursing process with
a newly hired nurse. Which of the following describes the planning phase of
the nursing process?

A. Identify the nursing diagnoses


B. Gather information if the patient’s problem has been resolved in the
evaluation phase
C. Review the patient’s history during assessment
D. Prioritize patient problems

27. Nurse jezyl one of the group leaders reviewed the steps of the nursing
process with the group. Which of the following date should the nurse identify
as objective data? (select all that apply)

I. Respiratory rate is 22/min.


Ii. Feels pain after a 10-minute walk
Iii. Pain is rated as 3 on a scale of 10.
Iv. Skin is pinkish in color, warm, and dry.
A. Ii and iii c. Iii and iv
B. I and iv d. I and ii

28. On the second day, the patient delivered an alive baby girl. She complains
of leg pain. The nurse took hold of the patient’s chart. Ponstan 500 mg every
4 hours prn for pain was ordered and was given. After 40 minutes, the patient
was relieved. What step of the nursing process should the nurse have
conducted?

A. Assessment c. Evaluation
B. Planning d. Intervention

29. According to the nursing process, which of the following actions the nurse
takes if the pain does not satisfactorily relieve?

A. Wait for more time for the pain reliever to take effect.
B. Collect additional data as to why the patient has not been relieve of
pain.
C. Teach the patient relaxation breathing techniques.
D. Refer to attending physicians.

30. The nurse trainor discusses the elements of documentation. Which of the
following refers to being comprehensive and timely?

A. Complete and current c. Organized


B. Accurate and concise d. Factual

Situation – patient ellie, a 28-year-old primigravida, is admitted to a


birthing center. She has been in labor with an interval of 5 minutes apart
from 10 hours now. Hypotonic contractions are observed by nurse nora. She
feels more pain in her back than in her abdomen, sonogram shows her fetus is
“borderline” large for gestation and in occipito-posterior position.

31. Nurse nora observes that the ellie’s uterine contraction are irregular in
frequency and short in duration. Ellie screams in pain during contractions.
Which of the following actions is considered best for the nurse platform?

A. Try to divert attention from pain.


B. Administer pain reliever as ordered.
C. Stay with the patient and offer her a back rub.
D. Document and report frequency and duration of contraction.

32. The physician is considering augmenting her labor with oxytocin. What
would make nurse nora question the use of oxytocin for patient ellie?

A. She had an amniocentesis performed during pregnancy


B. Her fetus is large for gestational age by a sonogram
C. Her membrane ruptured after only 1 hour of labor
D. Her blood pressure is slightly elevated above normal

33. Nurse nora notices patient’s uterine contractions are 70 seconds long and
occur every 90 seconds when assessing the frequency of her contractions after
she receives oxytocin. What would be the nurse’s first action?

A. Give an emergency bolus of oxytocin to relaxed the uterus


B. Discontinue the administration of the oxytocin infusion
C. Increase the rate of client’s iv infusion
D. Ask client to turn to her left side and breathe deeply

34. Nurse nora monitors the patient, knowing that which indicates an adequate
contraction pattern?

A. Three to 5 contractions in a 10-minute period, with


resultant cervical dilatation
B. Four contractions every 5 minutes, without resultant
Cervical dilatation
C. One contraction every 10 minutes, without resultant cervical dilatation
D. One contraction per minute, with resultant cervical dilatation

35. Which of the following nursing measures would the nurse least considers to
patient ellie with oxytocin drip?

A. Know how to recognize potential adverse reactions.


B. Administer oxytocin drug with caution
C. Monitor patient closely when infusing oxytocin
D. Inform patient about potential complications
Situation – miriam on one year of age, is admitted due to pneumonia. She has
iv antibiotics, antipyretic, decongestant and vitamins as medications. She
also is under oxygen therapy.

36. Nurse messy has been worried about miriam’s refusal to take her oral
drugs. How will she handle the situation?

A. Leave the child alone


B. Seek the help of the mother in giving the oral drug.
C. Mix the drug with milk to cover up the unfavorable taste.
D. Get angry with the mother and the child.

37. As one-year-old child, nurse messy understands the reason (s) why miriam
continuously refuse to take her drugs. It is because it is normal for her age
to .

A. Have separation anxiety.


B. Internalize the attitudes of others.
C. Utilize magical thinking.
D. Be negativistic in all matters.

38. The best way to administer oxygen on miriam is by .

A. Hood c. Incentive spirometer


B. Face mask d. Nasal catheter

39. For the iv antibiotic therapy of miriam, the most common gauge used for iv
cannula is gauge .

A. 20 c. 22
B. 24 d. 18

40. What important evaluation parameter should nurse messy observe that would
show improvement in miriam’s condition?

A. Absence of fever.
B. Absence of chest indrawing.
C. Respiratory rate of 45 beats per minute.
D. Respiratory rate of 55 beats/ minute

Situation - ashley a postpartum patient, who has delivered a stillborn wants


to leave the hospital without a physician’s order. The patient is still
hooked to an intravenous fluid(ivf) and is on closed postpartum monitoring.

41.to avoid liability, which of the following is an appropriate action by


nurse valerie?
A. Notify nursing supervisor of the patient’s plans to leave.
B. Arrange medication prescriptions at the patient’s preferred pharmacy.
C. Notify directly the attending obstetrician.
D.ask the patient about the transportation plans from the hospital.

42.nurse valarie informs patient ashley on the need for early ambulation.
Which of the nurse’s instruction on ambulation is incorrect?
A. Assist the patient from sitting to standing position.
B. Raise the head of the bed slowly to achieve sitting position of the
patient.
C. Allow the patient to rise from the bed to a standing position unassisted.
D. Assist patient to rise from lying to sitting position.

43.while waiting for a feedback from the nurse supervisor regarding the
[patient’s desire to go home, nurse valerie opted to check on the patient.
Upon entering the room, she discovers that the waste basket on fire. Sequence
the nurse’s actions in the options below.

I. Rescue the patient.


Ii. Activate the fire alarm.
Iii. Close the door to confine the fire.
Iv. Put off the fire with fire extinguisher.
A. Iv, ii, and i
B. I, ii, iii, and iv
C. I, ii, and iv
D. Ii, iv, and i
44. After the fire was put off, the patient was found to have absconded. What
is the ethico-legal responsibility of the attending nurse?
A. Autonomy
B. Nonmaleficence
C. Beneficence
D. Justice

45. Absconding is inevitable in any health care facility. Who will be informed
immediately if the patient found out the absconded?
A. Attending physician
B. Security guard on duty
C. Resident on duty
D. Nursing staff

Situation - catherine,5 years of age, is admitted to the pediatric ward due to


severe otalgia, fever and irritability. The mother informed nurse selma that
patient had upper respiratory infection three weeks prior to admission. The
admission diagnosis is acute otitis media (aom).

46. Nurse selma conducts her initial assessment on catherine. The patient
keeps on crying and constantly pulls her right ear. What is her most
appropriate action?
A. Request parent to carry the child
B. Take catherine’s vital signs.
C. Refer to the attending physician.
D. Assess the description and frequency of pain.

47. Nurse selma is preparing to administer ofloxacin eardrop on catherine per


doctor’s order. She needs to hold the bottle with her hands to warm up the
solution to prevent dizziness for_________.
A. 5-6 minutes
B. 1 to 2 minutes
C. 3-4 minutes
D. 6-7 minutes
48. After washing her hands and gently cleaning any discharge that can be
removed easily from the outer ear, nurse selma positions the child. Which of
the following steps follows?
A. Gently press the tragus of the ear four times in a pumping motion.
B. Gently pull the outer ear
C. Drop the medicine into the ear canal.
D. Keep the ear up for five minutes.

49. Based on her knowledge on otitis media, nurse selma recalls that children
are predisposed to aom due to the following risk factors, except ______.
A. Absence of breastfeeding
B. Swimming
C. Exposure to cigarette smoke
D. Poor hygiene

50. To promote drainage and reduce pressure from fluid, nurse selma’s nursing
intervention is to have the child assume any of the following positions,
except ___.
A. Tilt head to side if sitting up
B. Lie on the affected area
C. Put the pillows behind the head
D. Lie on the non-affected ear

Situation - nurse ester is rotated to the pediatrics ward. As such, she needs
to review the principles and concepts of human growth and development to
better appreciate her role as a professional nurse.

51. Being assigned to care for pediatric patients, nurse ester should remember
which of the following statements?
A. Toddler period ranges from 12 to 36 months
B. An infant’s tongue is smaller than adult
C. Early childhood period ranges from 3 to 7 years
D. Breast milk provides complete infant nutrition
52. While nurse ester was taking the temperature of the baby chooka, the
mother asked nurse ester when growth and development become more rapid. Her
answer should be, during at _______months of life.
A. Ten
B. Nine
C. Twelve
D. Eleven

53. It is vital for nurse ester to give concrete example of activities to


stimulate gross and fine motor development. Examples are, which of the
following?

1. Push/pull
2. Use of scissors and pencil appropriately
3. Poking straws into holes
4. Stand on tiptoes if shown first
A. 1 & 2
B. 2 & 3
C. 3 & 4
D. 1,2,3 & 4

54. According to the world health organization (who), suicide has become a
global phenomenon. When taking care of emotionally disturbed adolescent
patients, nurse ester should be alerted with warning signs which often occur
for at least one month before a suicide attempt, except____.
A. Increase in initiative
B. Verbalization of suicidal thoughts
C. Crying
D. Sleep disturbances

55. During one of the nursing rounds, the pediatric ward headnurse asked nurse
ester the inclusive ages considered as the transition from childhood to
adulthood but sometimes extending until college graduation. Her correct answer
should be _____.
A. 15 to 18
B. 12 to 16
C. 11 to 18
D. 13 to 18

Situation - in a birthing station, five postpartum mothers delivered 2 hours,


4 hours, and 6 hours ago, respectively. All of them are multigravida patients,
adalynn, the nurse educator opted to conduct health education on a postpartum
hemorrhage.

56. Nurse adalynn explains to the mother that early indication for hypovolemia
caused by postpartum hemorrhage is _____.
A. Increasing pulse and decreasing blood pressure
B. Altered mental status and level of consciousness
C. Dizziness and increasing respiratory rate
D. Cool, clammy skin, and pale mucous membranes

57. The nurse educator adalynn reviewed the risk factors for postpartum
hemorrhage for the mothers. Which of the following factors is not included
____?
A. Ruptured uterus
B. Uterine atony
C. Overdistended uterus
D. Retroversion of the uterus

58. During the normal postpartum course, when would the nurse expect to note
the fundal assessment that will be in line with the umbilicus?
A. Immediately after the delivery
B. 4 days after the delivery
C. When the client’s bladder is full
D. The day after the delivery

59. A postpartum patient asks nurse adalynn when she may safely resume sexual
activity. Which of the following information should the nurse tell the patient
on resumption of sexual activity?
A. In 2 to 4 weeks
B. At any time
C. After the 6-week physician check-up
D. When her normal menstrual period has resumed

60. Nurse adalynn discusses the possibilities of future postpartum hemorrhage


with the patients. Which of the following increases the absorption of vitamin
k?
A. Proteins
B. Carbohydrates
C. Minerals
D. Fats

Situation - during the nurse’s rounds, the head nurse noticed that the intake
& output sheets have not been filled up.

61.based on the findings, what should the head nurse do?


A. Ask the staff nurses the reason for the failure to properly fill up the
intake & output flow sheet.
B. Give the staff nurses first warning.
C. Conduct a needs assessment.
D. Review the orientation program.

62.the head nurse decided to coach her staff nurses. One of the questions she
raised was what fluids should be excluded in the i & o flow sheet.
The correct response should be, which of the following?
A. Intravenous fluids
B. Gelatin
C. Solid foods
D. Beverages

63.the head nurse emphasized to the staff nurses what not to be included under
the output list. The answer should be, which of the following?
A. Drainage from tubes
B. Solid/hard feces
C. Urine
D. Vomitus

64.the best time to record the intake & output is ______________.


A. During endorsement
B. After endorsement
C. Right before endorsement
D. Any time before duty

65.a patient’s i & o is vital for patients with chronic heart failure. The
main purpose of recording accurately the
I & o of such patient is to _________.
A. Determine is client is improving or not
B. Find out if there is still water retention in the interstitial cells
C. Detect cardiac overload
D. Determine weight gain/loss

Situation - the group of nurses assigned in the delivery room is interested in


conducting a study on the experiences of pregnant women in labor. They are
thinking of qualitative research.

66.in the presentation of results of qualitative research, the nurse research


uses as a reference in the write-up _____ person.
A. First
B. Second
C. Fourth
D. Third

67.nursing, as a human science, deals with the critical and fundamental


differences in attitude toward their respective phenomena. Which of the
following is an aim of human sciences?
A. Construct prediction
B. Seek causal explanation
C. Sets control
D.makes meaningful interpretation

68.the group was observant as to the activities taking place in the delivery
room. One of the activities involved social processes, which can be better
explored. Which of the following qualitative research method should be used?
A. Grounded theory
B. Historical research
C. Descriptive phenomenology
D. Case study

69.after the data analysis of their study, experiences of pregnant women in


labor, they returned to the participants to determine the accuracy of the
emerged themes. Which criteria of trustworthiness in the group doing?
A. Confirmability
B. Credibility
C. Transferability
D. Dependability

70.the group used an audio record to capture what transpired during the
interview. After the transcription, which of the following action is
appropriate for the group to do with audiotape?
A. Keep the audiotape in a vault and dispose of it a year after.
B. Submit the audiotape to their research adviser.
C.throw it in the trash bin immediately after it was used.
D.post the recording on their university research website for other to listen.

Saturation - marie ob-gyn head nurse, conducted an in-service program on staff


development.

71.head nurse marie discussed that the most frequently neglected area in
management is _____.
A. Managerial knowledge
B. Professional development
C. Clinical skills
D. Successful communication

72.a critical component of the supervisory process is delegation. Which of the


following in the most empowering to staff?
A. Effective delegation does not require nurses to know the abilities and
weakness of their staff.
B. Delegation frees the manager to do other tasks while empowering staff.
C. Delegation fosters the responsibility of staff while increasing
professional growth.
D. Delegation starts at top management down to subordinates.

73.head nurse marie discussed negotiation. The focus of negotiation is to


create a___________.
A. Soothing situation
B. Third-party consultation
C. Trade-off
D. Win-win situation

74.supervision occurs after delegation. What is the primary purpose of


supervision?
A. Influences the organization’s approach in recruitment promotion, and
personnel evaluation.
B. Improves staff compliance with policy and procedures individual.
C.assigns appropriate work tasks to the best-qualified individual.
D.enhances the delivery of quality nursing care.

75.delegation involves the transfer of care to an individual. What is the best


criterion when delegating staff?
A. Responsibility
c. Flexibility
B. Adaptability
d. Competence

Situation - therapeutic communication promotes understanding between the


sender and receiver. Nurse gary should be abreast with the common therapeutic
techniques if he wants his nursing care to be effective and achievable.

76.when a patience says,” i am not sure if i should undergo colonoscopy or


not as i am scared. “which of the following is the most appropriate
communication technique that nurse gary should use?
A. Touch
c. Restating
B. Clarifying
d. Silence

77.when a patient “whenever i see my husband visit me, i feel depressed”.


Nurse gary says. “your husband depresses you? “the therapeutic communicate
is which of the following?
A. Restatement
c. Focusing
B. Focusing
d. Seeking clarification

78.when a nurse gary says to the patient, “tell me more about your experience
when you had the colonoscopy.”, which of the following therapeutic techniques
is nurse gary using?
A. Focusing
c. Encouraging elaboration
B. Clarifying
d. Restating
79.when nurse gary, “tell me more the experience. I wish to hear about.
“which of the following therapeutic communication techniques is nurse gary
using?
A. Restating
c. Open-ended question
B. Seeking clarification
d. Summarizing

80.when nurse gary tells the patient, “you will be wheeled in to the or and
will be hooked to man ivf where the anesthesia will be given intravenously.”
Which of the following therapeutic communication techniques is nurse gary
using?
A. Clarification
c. Giving information
B. Summarizing
d. Reflection

Situation - a pediatric patient, 12 years old is admitted to the private room


with a tracheostomy tube.

81. Since the staff nurse assigned to the patient does not have any
experience in caring for a patient with tracheostomy tube, who among the
following should not do the care?

A. Medical resident

B. Medical intern

C. Charge nurse

D. Mother of a child with care of tracheostomy tube experience

82. The otolarhyngologist arrives to change the tracheostomy tube, which of


the following should the nurse collaborate for the appropriate
equipment/supplies need in changing the tracheostomy tube.

A. Emergency department
B. Central supply unit

C. Anesthesia department

D. Operating room department

83. To assure that nurse mica will learn the proper way of caring for patient
with tracheostomy tube, the heanurse should collaborate with, who among the
following personnel for the training?

A. Assistant chief nurse clinical

B. Chief of unit

C. Asst. Chief nurse for education and training

D. Chief of clinics

84. The otolarhyngologist ordered a change for tracheostomy tube ties? Who
among the following should the doctor collaborate with?

A. Medical intern

B. Medical resident

C. Nurse aide

D. Staff nurse

85. The skill of suctioning using a single use catheter for tracheostomy is
more safely performed with which of the number of assistant?

A. Four

B two

C. Three

D. One
Situation - josophine a multiparous patient is admitted due to labor pains
which started an hour ago, during the vaginal examination, the nurse noted the
complete dilatation of the cervix and effacement is 100 percent, the patient
is in the true labor pains

86. Which of the following problems with labor and delivery is completed in
less than 3 hours?

A. Precipitous

B. Preterm

C. Induced

D. Prolonged

87. Patient josephine was referred to the physician, routine blood


examinations were taken, after reviewing the serum electrolyte levels an
isotonic intravenous iv infusion was prescribed. Which iv solution should the
nurse prepare ?

A. 5 % dextrose in water

B. 0.45 percent sodium chloride solution.

C. 10% dextrose in water

D. 3% sodium chloride solution

88. The patient during labor would anticipate some emotional support. Which of
the following nursing interventions should nurse sarah provide to keep the
patient calm?

A. Giving praise to her the sense of satisfaction regard quick labor


B. Support in maintaining a sense of control

C. Explanation of the effect of labor on the new born

D. Allowing the patient to express pain ang anxiety

89. Patient josephine ask why her labor is so much shorter compared to her
previous deliveries. Which of the following is the best response?

A. Consent of contraction was gradual

B. Multigravida has shorter labor.

C. Cervical lengthening was longer

D. Induction of labor was done

90. Nurse sarah reads the physician's prescription to administer


methylergonovine maleate( methergen) im after delivery. The rationale giving
this medication is which of the following?

A. Reduces the amount of lochia drainage

B. Prevents postpartum hemorrhage

C. Decreases uterine contractions

D. Maintains normal blood pressure.

Situation - jose 10 yrs old has bronchitis he needs oxygenation 4l/m per
doctors order.

91. The first standard step in oxygen therapy that the nurse should do is
which of the following.

A. Prepare the patient for oxygen treatment


B. Check the chart for orderedq flow rate, and oxygen delivery method

C. Gather all the equipments and supply

D. Assess patients condition

92. In planning for jose's oxygen therapy the nurse should consider which of
the following?

A. Need for a humidifier

B. Length the tubing determine the sge of jose

C. Determine the sge of jose

D. Manner of administering oxygen

93. The priority action of the nurse for jose due to oxygen therapy is
___________

A. Attach the humidifier and connecting tubing to the oxygen delivery device.

B. Connect the flow meter to the pipe in oxygen outlet.

C. Turn on the oxygen

D. Check the flow

94. What priority precautionary measure should be done by the nurse during the
oxygen therapy?

A. Limit visitors

B. Attach "no smokong" signage

C. Check humidifiers water regularly

D. Connect belt to oxygen tank


95. One evening, jose complained of dyspnea despite continue oxygen therapy.
What should be nurse initial intervention?

A. Give prn medication

B. Refer patient to the physician

C. Assess the latency of the tubing

D. Re-assess the patient

Situation - headnurse wilma has been encountering errors in documenting and


records management based on her reviewe of the nurses notes in the patients
chart, to solve the issue, she decided to conduct a lecture on proper nursing
documentation and management of records.

96. At the start of her lecture, headnurse wilma ask the purpose of the
nursing process, which of the following purposes is the correct answer?

A. Reduce the number of forms of the chart

B. List the patients health problems

C. Record the patients progress

D. Provide the confidentiality of the chart

97. One of the staff nurses was asked about the principles to be observed when
charting patients progress accurately, which of the following principles would
be the correct answer?

A. Statements are qualified by the use of "seems" and "appear"

B. Assumptions and conclusions are reported

C. Specific and definite words or phrases are used


D. General statements and measurements are used

98. Which of the following is not a characteristic of charting?

A. Complete

B. Subjective

C. Objective

D. Accurate

99. During nursing endorsement, the kardex used. Which of the following
statement is not correct? It is____________

A. Kept up to date.

B. A quick reference to current information about the client.

C. Consists of folded card for each patient.

D. Part of the medical record.

100. A sample of an error in charting was shown by headnurse wilma. Which of


the following is the correct solution to remedy the error?

A. Recopy the sheet and destroy the original sheet.

B. Use a single line to cross out the error, then write the date, time and
sign the correction made.

C. Use correction fluid to erase the error.

D. Use eraser to remove the wrong entry.


Nursing practice iii: care of clients with physiologic and psychosocial
alterations, part a

Situation - nurse g is assigned in the morning shift of the post- anesthesia


care unit (pacu) and has a patient for admission.

1. The initial priority assessment performed by the nurse, when admitting a


patient in the unit after abdominal surgery is to check for _______________.

A. Surgical site for drainage and hemorrhage


B. Skin color and temperature
C. Responsiveness to painful stimuli and noise
D. Respiratory function and airway

2. How should the nurse position the patient who is in a somnolent status and
still under the effect of anesthesia?
A. Supine position with head bed slightly elevate
B. Prone position with a pillow under the abdomen
C. Semi-fowler’s with the head turned to the right
D. Left lateral position with a pillow supporting the head

3. When a patient has bleeding after surgery, the pacu nurse, expects which
color if coming from arterial source?
A. Darkly-colored , blood flows fast
B. Bright red and spurts with the heartbeat
C. Slow, dark-colored, generally ooze.
D. Pinkish colored-slowly flowing

4. During the immediate post-operative phase, the primary goal of the health
care team is to maintain ventilation. Which of the following situations should
be observed as a result of hypoxemia?
A. Excess carbon dioxide in the blood
B. Increasing ammonia in the blood
C. Decreased oxygen saturation
D. Reduction of blood ph

5. When a patient develops a temperature of 39.8 degree centigrade after an


abdominal surgery with an ongoing blood transfusion, the pacu nurse should
notify the surgeon as this may indicate___________.
A. Abdominal tissue injury
B. Ongoing potential infection
C. Post –anesthesia drug reaction
D. Allergic reaction from blood transfusion

Situation - nurse sandy is a member of the research team in a cardiovascular


health facility. One of the research priority problems in her unit is on the
“effects of personalized care management strategies on stress after an acute
myocardial infection (mi)

6. Nurse sandy selected a framework which focuses “ that humans are in


constant relationship with stressors in the environment and that nursing is
keeping the patient’s system adjust to wellness which is best reflected in
_________.

A. Parse’s human becoming theory


B. Neuman’s adaptation model
C. Peplau’s psychodynamic theory
D. Swanson’s theory of caring

7. What type of sampling while nurse sandy adopts if when every third (3rd)
patients with mi are selected after a random start?
A. Simple random
B. Systematic
C. Stratified
D. Cluster

8. Nurse sandy distributed the coded questionnaires to the respondents of the


study without their names on it. What ethical safeguard is being employed in
this practice?
A. Confidentially
B. Truthfulness
C. Trustworthiness
D. Anonymity
9. When the findings of the study can be applied to all patients with mi under
stress patients in the whole philippines, it satisfied which of the following
criteria in research?
A. Reliability
B. Validity
C. Generalizability
D. Transferability

10. Nurse sandy has completed the study and started to make a report to the
research department. A written brief summary is submitted which is known as
_________.
A. Narrative
B. Abstract
C. Critical appraisal
D. Monograph

Situation – communication is a basic competency needed by the nurse in the


health care delivery system as a means of building relationships with patients
and significant others.

11. Mrs. Lim, 52 year old has been in the hospital for almost a week. She
approached nurse alma who was doing her rounds and said “ i wish that my
children will visit me today” your best response to the patient is ________.

A. “do you have their cell phones, if you want i can call them”.
B. “just relax, i am sure they will come and visit you today”
C. “you sound to be lonely, do you want me to be with you for a while”
D. There is so much traffic, anyway it is not yet time for visiting hours”.

12. Nurse linda observed that her patient, who was just admitted for abdominal
pain, looks tense and quite restless. The patient tells the nurse “i am
afraid to undergo the physical examination”. The best response of nurse linda
is ________.
A. “i will inform the physician that you are afraid to undergo the physical
examination”
B. “there are several patients waiting for me, the examination has to be done
now”
C. “i will call your relative to be with you, while you are being examined”
D. “the examination will not hurt, would you like to sit down first and
rest”

13. Ms. Cynthia, a college student is being admitted on and off in the
hospital for bronchial asthma verbalizes to the nurse “do you think i should
stop attending my classes? What is the best response of the nurse?
A.” Well to me, that is a good decision for now especially with your asthma.
B. You said, you are thinking of stopping form attending your classes?
C. “let us consult your physician, which one is better for you”
D.” Do you feel that is a good decision for you and your parents”

14. An elderly patient looks depressed, tells the nurse while her blood
pressure is being taken, ”go away, leave me alone, i don’t want to talk to
you anymore”. The nurse best response is _________.
A. “don’t say that, otherwise i will not come and see you anymore”
B. “i will be back in an hour, by that time, you might want to talk to me”
C. “ if you need me, just inform your watcher for some help.”
D. “don’t say that, i will just attend to the other patients in the ward”

15. Patient alma, is receiving insulin injection for her diabetes. She tells
the nurse “ i don’t want to have that insulin injection anymore.” The most
therapeutic response of the nurse is __________.
A. You need the insulin injection now, i will refer you later will not be
controlled.”
B. Just have your insulin injection now, i will not, i will refer you later to
your attending physician.”
C. You physician will get mad, if you will not have your insulin injection.”
D. Let us sit down, i will explain you you, why you have to receive the
insulin injection.”

Situation – nurse lina is assigned in the medical ward and has admitted
patient sonia, a 52 year old accountant, who was suspected to have of crohn’s
disease by her family physician.

16. Nurse lina should assess the patient for the presence of _______.

I left lower quadrant abdominal pain


Ii diarrhea, unrelieved by defecation
Iii abdominal tenderness and spasm
Iv increase in weight
V excessive fat in the feces
A. Ii & iii
B. Ii, iii & iv
C. Ii, iii, & v
D. I & ii

17. The physician in-charge of the patient ordered laboratory and diagnostic
studies in order to confirm its diagnosis. This of the following examination
is conclusive of the presence of crohn’s disease?
A. Upper gi series
B. Proctosigmoidoscopy
C. Barium enema
D. Abdominal radiography

18. During ms. Sonia’s hospitalization, she had fever, loose bowel movement
with foul smelling odor with abdominal pain. Considering these manifestations,
nurse lina should monitor the patient on what specific condition?
A. Hypocalcemia
B. Thrombocytopenia
C. Hyperbulemia
D. Hypokalemia

19. Ms. Sonia was ordered by her physician to have total parenteral nutrition
(tpn) to improve her hydration, skin turgor and improve the overall well being
of the patient. Which of the following nursing measures should nurse lina
implement to facilitate positive patients outcome?

I weighing the patient daily


Ii measure intake and output
Iii provide relief of pain & discomforts
Iv monitor iv infusion rate daily
A. I, ii & iii
B. I & iii
C. I, ii, iii & iv
D. I & iv

20. The patients is preparing to go home in few days time. What specific
health teaching on home parenteral nutrition (pn) should be given considering
the level of education of the patient. These include the following except
_________.
A. Use of aseptic technique in changing the dressing.
B. How to store & preparing the solution properly.
C. Setting up of the infusion pump safely.
D. Flushing the parenteral nutrition line with antibiotics

Situation - in today’s nursing practice, roles have been expanded to include


legal responsibilities and accountability in workplace.

21. Nurse lina is on duty at the er and has been very busy that morning
resulting to the administration of penicillin injection which is ordered
to another patient. With this error, the nurse can be charged of ______.

A. Malpractice

B. Negligence

C. Assault

D. Battery

22. Transcription of doctor’s order is a nurse’s responsibility to put the


order into action. Which of the following principles of medication safety
is not considered to belong to transcription error?

A. Illegible handwriting of the physician.

B. Misinterpretation of the directions ordered.

C. Use of unapproved abbreviation in the chart.

D. Wrong route of medication administration.

23. In order for nurses to encounter legal problems in drug administration


which of the following 7 rights of drug administration, should be implemented
under the category of “right drug”? Read the label of the drug ________.

I. Three (3) times.

Ii. Before removing from the shelf.


Iii. Before measuring actual dose.

Iv. Before opening unit dose -container.

A. Ii & iii c. I, ii & iii

B. I & ii d. I, ii, iii & iv

24. Nurse edna admits a patient from the er to the medical unit. The patient
is very restless with iv lines and a urinary catheter. She was put to bed
and the nurse applied a body restraint without the doctor’s order. Nurse
edna’s action can be liable for ______.

A. Invasion of privacy c. Battery

B. Non- maleficence d. Neglect

25. Ms. Cruz is a supervisor of the hospital on night shift. Several nurses
did not report due to jeepney strike. When the supervisor is executing
proper allocation of nurses to the priority wards to address this
concern, she is observing what type of principle?

A. Beneficence c. Justice

B. Non- maleficence d. Fidelity

Situation – mr. Jc 50–year-old is having episode of bronchial asthma and was


examined in the er by the physician and ordered to be admitted.

26. Nurse tita admitted the patient and was observed to be dyspneic. Breadth
sounds are diminished upon auscultation. Which initial nursing action should
she implement first?

A. Position him on bed in semi- fowler’s position.

B. Administered oxygen therapy as ordered.

C. Instruct patient to nebulize self if available.

D. Administer bronchodilator as ordered.


27. Based on the health history, the patient is working in a cement factory.
This occupational hazard can bring about asthmatic trigger as a/ an
_______.

A. Contactant c. Infiltrant

B. Inhalant d. Ingestant

28. The physician ordered theophylline 300mg. Per day to relieve patient’s
asthmatic attack. Which of the following is not mechanism of action of this
drug?

A. Regulating the immune system of the body.

B. Relaxing the smooth tissue of the bronchial airways.

C. Reducing inflammation of the airways of the lungs.

D. Relaxing the skeletal muscle of the bronchial airway.

29. Select from the following potential adverse effects of theophylline, the
nurse should observe when the drug is taken orally_____.

A. Dry mucosal membrane c. Irregular heartbeat

B. Inability to sleep d. Increased blood pressure

30. The patient is preparing to go home with his bronchodilator drugs. He ask
the nurse, how will i know if the drug is effective? Her appropriate
response is, “ there will be an initial decreased in _______”.

A.allergic reaction c. Body temperature

B. Breadth sounds d. Wheezing

Situation - noel, a 67 year- old, male, anxious and restless with his
condition, was admitted by nurse chona in the medical ward due to chronic back
pain, general weakness and difficulty of breathing. He was attended by the
physician and was tentatively diagnosed with impending respiratory
insufficiency secondary to hypoxia.

31. It is important that nurse chona records accurately the restlessness


caused by pain and that of hypoxia. Which of the following should be
recorded as the restlessness caused by pain?

A. Difficulty of breathing.

B. Increased respiratory rate and blood pressure.

C. Increased heart rate.

D. Increased perspiration and change of position.

32. Nurse chona read in one nurse’s notes chart this documentation: “
refused to eat and fell from bed”. Which of the following is lacking in
this documentation?

A. Time of complaint, foods missed and reaction on fall incurred.

B. Refferals made on fall, medications given and reasons of falling.

C. Contents of complaints, reasons of refusing meal and nature of fall.

D. Time of eating, medications for back pain and intense of pain.

33. Charting are important documents that are used in court proceedings nurse
chona should take note that the following entry recorded would be most
defensible in court?

A. Large bruises on thigh upon assessment.

B. Patient fell out of bed when reaching out for medication.

C. Burn on the back area observed upon auscultation.


D. Patient drunk when seen in emergency room.

34. Which of the following charting rules will keep the nurse legally safe?

I. Documenting worries and all concerns as verbalized by the patient.

Ii. Charting at the end of the shift only.

Iii. Discussing of recorded cases and diagnosis of the patient.

Iv. Recording all information verbalized by patient and family.

A. Iii, iv c. I

B. I, iii d. Ii

35. Nurse chona saw patient noel reading hid own chart and questions the nurse
why(-) smoking and (-) liquor was recorded when he does not smoke and drink
alcohol? What is the initial explanation of nurse chona on the record?

A. Get the chart from and reprimand him from reading the chart.

B. The sign of negative before and the word means he is not drinking alcohol
or smoking cigarettes .

C. Ask patient noel to apologize reading the chart.

D. Tell noel that alcohol and liquor are important factors in diagnosing his
ailment.

Situation – ethical dilemma is becoming a common scenario in the health care


delivery system brought about by the advance technological changes on health
care.

36. Which of the following is the example of an ethical dilemma?


A. A nurse providing demonstration of the possible risk if a chest tube
drainage of a patient is pulled out.

B. Family members having ambivalent feelings whether their father will undergo
amputation of a diabetic leg.

C. A nurse is overheard by the daughter that if her mother will not take her
insulin injection she will not serve her food tray.

D. The physician signs the medical directives of a terminally- ill patient


admitted in the hospital with his spouse.

37. A 70- year- old has been rushed to the hospital due to bradycardia and
palpitation. The physician suggested that pacemaker be inserted to correct
the symptoms. The patient voluntarily decides not to have the pacemaker
inserted. This is respected by the family. This is an example of what ethical
principle?

A. Beneficence c. Fidelity

B. Autonomy d. Justice

38. The er nurse clarifies the doctor’s prescription on the dose of the pain
medication base on the pain scale assessment and patient’s age. Which
ethical principle is applied?

A. Fidelity c. Justice

B. Truthfulness d. Non- maleficence

39.nurse pat promised to a post- surgical team, that she would come back to
assist in his ambulation after carrying out the physician’s order. This
follows the principle of______.

A. Beneficence c. Autonomy
B. Justice d. Fidelity

40. When a nurse supports the welfare of the patient in relation to health ,
safety and personal rights, the ethical principle followed is ______.

A. Responsibility c. Autonomy

B. Accountability d. Advocacy

Situation - nurses dina is caring for several patients in the medical unit
mostly sufferings from endocrine disorders evelyn, is a 45 years old,
government employee who has been diagnosed by their doctor to be suffering
from graves disease. Based on her health history she has been having insomia,
, palpitation and heat intolerance.

41. A classic finding that nurse dina would expect to find in addiction to
the above complaints which is specific to her condition is ____

A. Decreasead libido c. Infertility

B. Exopthalmos d. Hoarseness of the voice

42. Select the two (2) primary laboratory test findings ordered by his
physician that will confirm that patient evelyn has graves diseases.

1 decreased tsh levels


11 elevated free thyroxine levels
111 increased tsh levels
1v. Decreased free throxin levels

A.11 &1v c. 1 &11


B. 111 &1v d. 11 & 111
43. Choose from the listed drug therapy that should be administered to ms.
Evelyn in order to inhibit the synthesis of thyroid hormones and block the
conversion of t3 and t4___
a. Adrenergic blockers (inderal)
b. Radioactive iodine (rai)
c. Sevothyroxine (synthroid)
d. Propyl thiouracil (propyl thyracil)

44. While ms. Evelyn is being prepared physically and physiologically for
possible thyroid surgery, her nutritional needs was also being met wit a die
that is__

A. High fibrous, low protein, high cho

B. High caloric, high cho, low protien

C. High caloric, high cho, and protien

D. High protein, low vitamin a &c

45. Ms evelyn underwent thyroidectomy she complained of numbness of the


finders, toes and twitching of the mouth, this is likely caused by __

a. Thyrotoxic crisis c. Hypo thyroidism

b. Hypo parathyroidism d. Infection

Situation - mr. Santi a salesman was admitted because of severe abdominal


pain. Based on the data gathered, he smoke and drink 3-4 bottles of beer every
day. He appeared jaundice, dyspeic with enlarged abdomen. The physician
suspected that he has a liver disorder abdomen. The physician suspected that
he has a liver disorder and beginning esophageal varice.
46. Which of the following clinical manifestation is not included in client
with hepatic disorder?

a. Indigestion c. Nausea and vomiting

b diarrhea d. Abominal bloating

47. Liver function test was ordered. In a damaged liver the following tests
will reveal

1. Prothrombine time is prolong.

11. Albumin level is ordered

111. Globulin level is decreased

1v. Amonia level is decreased

A. 1 and 11 c. 1 only
B. 1.11 and 111 d. 1.11.111 and 1v

48. Mr. Santos esophageal varices bleed. His doctor ordered sengstaken-
blakemore tube to stop bleeding. Two hours after, the client develop
difficulty of breathing. What priority action should the nurse undertake?

a. Monitor vvital siign

b. Asked client to take a deep breath

c. Defflate the esophagel baloon

d. Call the physician at once

49. In client with esophagel varices which of the following doctors order do
you anticipate?

a. Monitoring progression of anxiety


b. Observe signs of epistaxis

c. Monitoring urine output

d. Preparing client for blood transfusion

50. Which of the following medication is likely to be given to quickly stop


bleeding from esophageal varices____

a. Neperidine c. Aldactone

b. Furosemide d. Vasopressin

Situation - mr. Leo, an insurance agent was ordered by his physician to be


admitted to the hospital for coronary artery bypass graft (cabg) due to three
vessels blockage.

51. As a nurse, you are aware that cardiac surgery is a source of stress to
the patient and family. Which of the following strategies should the nurse
implement first to overcome this stress?

A. Identity ccoping mechanism helpful to the patient and family embers

B. Recognize fears and concern regarding surgery and future health status.

C. Explore support system available during the entire hospitalization period

D. Reinforced understanding of the surgical procedure, hospitalizaton and


recovery

52. Mr leo, underwent an invasive diagnostic test to determine the location of


the blockage which is needed for his cabg. Thi procedure is ___

A. Cardic ct scan

B. Carotid doppler
C. Magnetic resonance imaging

D. Cardiac catherization

53. The nurse is teaching breathing exercise to mr. Leo at what phase of the
peri- operative care is this best performed?

A. After surgery when an in inside the recovery room

B. Immediately after he has signed the informed consent

C. During the briefing period prior to the surgical procedure.

D. When he is back to his room from the recovery unit

54. Which of the following is a blood thinning drug and temporarily stopped by
the physician prior to cabg due to possible bleeding?

a. Prodexal c.aspirin

b. Ibuprofen d. Toradol

55. The nurse formulate a nursing diagnosis decreased cardiac output r/t blood
loss which of the following is the highest priority nursing action?

a. Auscultate for heart sound and rhythm

b.assess peripheral pulses, pedal , tibial, and radial

c. Monitor ekg pattern for cardiac dysrhythmia

d. Assess arterial blood pressure every 15 min. Until stable

Situation - venus, 32 year old a government employee has been having abdominal
pain which was on and off for almost 6 months. She has been having flatulence,
and recently lost weight because of vomiting. She consulted their office
physician and she was suspected to be suffering from chronic pancreatitis. She
was advice to be hospitalized for further work-up.

56. You are the admitting nurse when ms. Venus arrived in the hospital. When
doing a comprehensive pain assessment, you should conduct the procedure during
the ___

A. Evaluation of nursing pain management

B. Initial contact with the patient

C. Course of pain management

D. The time the physician instructed you to do

57. When a patient is having pain due to pancreatitis, you expect that the
pain is located in the ___

A. Hypogastrium, right upper quadrant of the abdomen radiating to the left


lumbar area

B. Epigastrium, right upper quadrant of the abdomen radiating to the left


lumbar area

c. Hypogastrium, left upper quadrant of the abdomen radiating to the left


lumbar area

D. Epigastrium, left upper quadrant of the abdomen radiating to the left


lumbar area

58. You are aware of several manifestation when a patient is in pain. Which
of the following is a behavioral response to pain?

A. Changes in skin color

B. Increase in blood pressure


C. Depth in respiration

D. Moaning and grimacing

59. When a patient is ordered corticosteroids, which of the following drugs


will produce therapeutic effects of reducing pain?

a. Spironolactone c. Atropine s04

b. Diazepam d. Prednisone

60. When severe vomiting occurs in this patient, it results to what particular
condition?

A. Hyperkalemia c. Alkalosis

B. Hypocalcemia d. Acidosis

Situation - collaboration and team work are critical to the success of health
care operations

61. Which of the following statements is incorrect of collaboration in


health care setting?

A. Trust and respect are core values of a collaborative organization.

B.a shared vision is essential for collaboration in any health care


operations.

C. Successful conflict resolution can help collaborative teams overcome


differences.

D. Inter professional collaboration multidisciplinary collaboration can be


used interchangeably.
62. Nurse lyn received an order from attending physician of patient pillar who
was having a severe abdominal pain of almost a double dose of morphine so4 the
nurse consulted her and tell the manager and the pharmacist regarding the
order which was also questioned. The nurse called the attention of the
physician regarding the dose of the drug and changed the order to an
acceptable level what relevant principle of collaboration is applied in this
case?

i.shared vision

ll.respect and trust

lll.communication

lv.interpersonal relationship

a.i,ll,lll, and lv

b.lll, and lv

c.l and ll

d.i,ll, and lll

63. In a tertiary hospital where gio works as unit manager, an


interdisciplinary team model is adopted on collaboration of care, with this
type of model how is decision making made?

A. Shared responsibility of the group for the problem solving final decision

B. One person makes the final decision for the treatment.

C. Partnership with the patient and team for the final decision.

D. All members work together for both alternative and final decision.
64. Nurse ime, is a member of the quality assurance team of the hospital and
has been always rated as very assertive which of the following is not a
characteristic of an assertive person?

A. Intervene with the situation calmly and confidenty

B. She stands up with what she believes and push control on others

C. Articulate clearly the importance of nursing perspective.

D. Use “i” when stating thought and feeling and “you” when persuading
others.

65. A patient is going for a coronary artery bypass graft (cabg) due to a 4
black arterial blood vessels.a surgical team has been formed with the cardiac
surgeon as the head who is a member of the health team that prepares the
preoperative orders for the patient?

a. Cardiologist

b. Anesthesiologist

c. Surgeon

d. Medical internist

Situation - roy, 65 years old came to the outpatient clinic due to


dyspnea,fever and on and off productive cough he smokes one pack of cigarettes
per day for the last 30 years.the doctor ordered sputum examination, chest
x-ray and blood culture.

66. In collecting sputum specimen the nurse should it should instruct roy to

a. Breath slowly cough and expectorate into the specimen

b.breath deeply and cough expectorate into the sputum container


c.cough and expectorate saliva into the specimen container

d. Cough and expectorate into the specimen container

67. The patient’s diagnostic test revealed he was positive for bacterial
pneumonia the most like the causative organism of this type of pneumonia is

a. Legionella pneumoniae

b.mycoplasma pneumoniae

c.streptococcus pneumonia

d. Hemophilus pneumoniae

68. The nurse did an admission procedure the best position to be assumed by
roy is

A. Left lateral position with the affected side inferior

B. Sime fowler position at least 30 degrees

C. Dorsal position with pillows under the chest

D. Lying on his side the affected side of the lungs should be superior

69. When a patient is diagnosed to have pneumonia the breath sound detected by
the nurse on auscultation of the affected area would be

A. Wheezing sounds

B. Stridor

C. Fine crackles

D. Deep and low-pitched breath sounds


70. When there is respiratory depression resulting from drug overdose the
nurse have to watch for which of the following?

A. Hyperventilation

B. Tachypnea

C. Biot’s respiration

D. Bradypnea

Situation - ms bel 66 years old consulted the opd due to changes in energy
level fatigue and not able to tolerate performance of activities of daily
living the physician suspected that she has hypothyroid disorder and advised
admission for further work-up.

71. When a patient is with hypothyroidism the assessment findings of the nurse
that is not present is

a. Brittle nails

b.hair loss

c.dry skin

d. Fine tremors of hands

72. In assessing the thyroid gland for size, shape symmetry, consistency and a
presence of tenderness, the most appropriate examination modality is

a. Auscultation

b.percussion

c.palpation

d.inspection
73. After several thyroid diagnostic tests,ms bel was ordered to take thyroid
hormone( levothyroxine). Which of the following nursing actions is not
advisable in administering this drug? The nurse should give?

a. A single dose daily before breakfast.

b.a single dose daily after breakfast

c.it with a full glass of water

d.it without mixing with fruit juices

74. In monitoring the effectiveness of the drug therapy which of the following
is not expected as a positive patient outcome?

a. Regular bowel function

b. Excessive sweating at night time

c. Participates in self care activities

d. Metabolism returns to normal

75. The nurse in developing a nursing care plan for ms bel if the nursing
diagnosis is “activity intolerance related to fatigue and depressed cognitive
process which of the following is an appropriate intervention?

a. Increase mobility through early ambulation

b. Allow self care activities with active exercises

c. Space nursing activities to promote rest and sleep

d. Avoid a stimulating interesting conversation

Situation - resources in healthcare delivery system are one of the challenging


concerns of nursing leadership in the country today. Nurse gayle is a unit
manager in a tertiary hospital and conducts meeting regularly every two weeks
to her staff to address priority affecting their services.

76. Which of the following statements indicate an effective communication


technique used by the unit manager to her staff?

A. “we need to improve our nursing services otherwise top management will
take over”

B. “let’s limit requesting supplies and equipment our budget for our
promotion might be affected”

C.” There are a lot of redundant positions in our unit there is a need to
retrench some stuff”

D. “we need to discuss strategic approaches to facilitate delivery of nursing


services with less expense on our consumers”

77. For the past six months several nurses are resigning some have verbalized
in the exit interview that there are not happy anymore which of the following
descriptions manifest best a nurse who has job satisfaction?

A. Competitive, self centered euphoric

B. Empowered, enthusiastic,ethical

C. Loner,egoistic,reactive.

D. Outgoing ,sensitive ,competitive

78. Delegation is a critical component of leadership and governance which of


the following empowering activities should the unit manager applies in
delegation?

A. In delegation facilitating professional growth and development of a staff


is necessary.
B. When delegating i responsibility to a nursing staff modification of
standards of care is permitted.

C. Delegation of responsibility in patient care has to start from top


management to the staff nurse level.

D. Delegation requires responsibility with corresponding accountability of


staff nurse.

79.nurse gayle is guided that the initial step of delegation is?

A. Demonstrate the task and let the staff continue the next activities

B. Assess the capability of the staff change him if not performing

C.determine the competency level of the staff-for the task being given

D.explain the the task to be done with the companying job description

80. The unit manager is planning to take her regular official business leave
for the year-she has written a letter of recommendation to her immediate
supervisor for her assistant unit manager to assume her position while she is
on leave. This is example of delegation by______:

A.rank

B.authority

C.succession

D. Authority

Situation – nurse adela is on duty in the medical unit and has two patients
for discharge.
Eliza, 38 years old diagnosed with diabetes mellitus and on insulin therapy
for the first time and aileen, 42 years old who had myocardial infarction (mi)
81. When preparing a teaching plan for patient eliza, the following are the
instructions provided to patients with diabetes mellitus. Which one of the
teaching plan is considered not a priority concern when discussing the list
with the patient?
A. Dient and genetic counselling.
B. Exercise in extreme heat and cold.
C. Regular exercise, diet and medications.
D. Monitoring of blood sugar and urine ketones.

82. Eliza was taught by nurse adela on how to administer self-injection


insulin and rotation sites with the use of the chart every day till her
discharge.the following is the procedure for the self – injection of insulin.
Which of the following are the correct sequence in the self administration
of the drug?

I. Inject the insulin, push the plunger all the way in


II. With one hand stabilize the skin by spreading a large area
III. Pull the needle straight out of the skin & press cotton ball over
injection site
IV. Pick-up syringe with the other hand, hold and insert needle to the
skin.
V. Dispose syringe in the hard plastic container.

A. I, iv, iii, & v c. Iii, ii, i, v &


iv
B. Ii, iv,i, iii & v d. Iv, iii, i, v &
ii

83. The doctor ordered a low- saturated fat, low- cholesterol diet to patient
eileen who was also going home. From the following list of foods, which meals
has to be included in the instructional plan of nurse adela to her patient?

A. Pork steak, mixed vegetables with butter and cheese.


B. Hamburger, macaroni salad and milk shake.
C. Fried chicken, green beans, and skim milk.
D. Baked fish, green beens, coffee

84. Considering that patient eileen is a post mi patient, she was taught by
the nurse on home exercises on leg movements while resting on bed. The
expected goal of this intervention is to______:
A. Facilitate better digestion is to _____.
B. Prevent stasis of urine and stone formation
C. Facilitate circulation for skin integrity
D. Prevent venous stasis in the lower extremities

85. The resumption of sexual activity is an important activity that has to be


included in the home instructions to be given to patient eileen together with
her husband.which of the following is the safe period for an complicated mi to
resume sexual activity?

A. One month after mi


B. 12 to 14 days after mi
C. Two months after mi
D. 7 to 10 days after mi

Situation – mo, a 22 –yr old nurse graduate, passed the november 2018
philippine nurse licensure examination before her birthday which is november
16. The scheduled oath-taking ceremony was set on january 6, 2019.

86. To obtain her license to practice, she must do the following, except:

A. Must take the oat to any government official


B. Must register in the registration division of the prc
C. Must take the oath of professional before any member of the prbon
D. Must be issued a certificate of registration(cor) and profession
identification card (pin)

87. Nurse merle has to renew her license on before ______.

a. January 6, 2021 c. November 16, 2021


b. November 16, 2022 d. January 6, 2022

88. For the nurse merle to continually practice nursing, she must satisfy the
requirements set by the prc and comply how many units of cpd upon renewal?

A. 45 units c. 25 units
B. 10 units d. 15 units

89. After 3 years being assigned in the operating room, merle in interested to
actively join which appropriate professional organization?
A. Ornap
B. Nlgn
C. Adpcn
D. Apo

90. What would merle do to keep abreast with the latest trends in
peri-operative nursing?

A. Attend training and seminars


B. Perform researches
C. All of these
D. Pursue graduate studies.

Situation – a computer analyst of a company, was rushed to the emergency room


due to
Abdominal pain, nausea and vomiting, ascites and shallow breathing due to
enlarged
Abdomen. The physician suspected that the patient is suffering from
peritonitis and was
Advised to be admitted for further workout.

91. In assessing a patient suffering from peritonitis, which of the


following manifestation is not likely to be present?

A. Rebound tenderness
B. Abdominal gas
C. Abdominal guarding
D. Abdominal regidity

92. The overall goals set by the members of the heath team once the patient is
admitted include which of the following?

I. Relief of the abdominal pain


II. Resolution of inflammation
III. Provision of normal nutritional status
IV. Prevention from complications

A. I, ii, iii
B. I & ii
C. Ii & iii
D. I, ii, iii, & iv

93. Diagnostic test were ordered to the patient which included: complete blood
count (cbc) and abdominal ct scan. The primary purpose for ordering ct scan is
to determine presence of_______.

A. Amylasse content
B. Bacteria
C. Fluid shifts
D. Abscess

94. Which of the following drugs do you expect be ordered by the physician for
severe acute pain?
A. Acetaminophen (tylenol)
B. Levorphanol (levo-dromoran)
C. Percodan (aspirin)
D. Codeine (ambenyl)

95. The physician ordered the patient for paracentesis. Which of the following
pre-procedure nursing actions should the nurse perform except______.

A. Place him in upright position on the edge of the bed.


B. Place him in low-lying position with knees straight
C. Check for the signed consent form
D. Instruct patient to urinate to empty bladder

Situation – jen is an active member of the quality assurance (aq) team in a


health facility where the major role is setting quality standards.

96. Which of the following statements is true of quality improvement?

A. Focuses on the organizational structure than patient care.


B. Continually improve every process in the organization.
C. Centered on people rather than processes.
D. An approach that is externally driven by the stakeholders.

97.as a qa nurse, which of the following can nurse jen adopts as a concurrent
evaluation of patient’s outcomes in the unit?

I. Direct observation of patient’s condition.


Ii. Patient’s interview in the unit.
Iii. Face to face interview with nursing staff.
Iv. Nursing audit from the patient’s chart.

A. Iii & iv
B. I, ii, & iii
C. I, ii, iii,& iv
D. I, & ii

98. Nurse jen, distributed the patient satisfaction form to the admitted
patients in the hospital, this tool is in complience to the ______ quality
standards.

A. Human resource
B. Organizational structure
C. Patient care
D. Legal – ethical

99. The qa team was alerted by the administration, that they have to
anticipate for a possible earthquake in the forthcoming days. As part of the
safe practice environment (spe)
Standards, which of the following are to be given priority attention by the
team?

I. Electrical wires
II. Combustible materials of the building
III. Entry , exit
IV. Infected waste materials

A. I, ii, iii, & iv


B. Ii, . & ii
C. I, & ii
D. I, ii, & iii

100. The qa team has bombared by complaints of patients on their long waiting
period in the opd , before the health care professionals are examining them.
In response to this concern, which type of quality assessment should the team
implement?

A. Outcome evaluation
B. Process evaluation
C. Ongoing evaluation
D. Structure evaluation

Nursing practice iv: care of clients with physiologic and psychosocial alterations,
part b

Situation - research is a vital endeavor nurses must engage into in order to


contribute to nursing science

1. When the nurse researcher collects data at more than one point over an
extended period, which design is applied?
A. Cross-sectional
B. Time-related
C. Time sequenced
D. Longitudinal

2. If a research study involves an intervention and “blinding” which


research design is being referred to?
A. Non-descriptive
B. Phenomenological
C. Experimental
D. Descriptive

3. Which of the following statements is least descriptive of a qualitative


research design?
A. Researchers become involved.
B. Gather data from one collection strategy.
C. It is flexible and elastic.
D. Strives for an understanding of the whole strategy.
4. Qualitative researchers should choose their participants who can best meet
the objectives of the study. Who of the following best qualifies?
A. Cooperative person in the community
B. Those readily available thus convenient for the researcher.
C. Able to articulate and reflect on the phenomenon that they experienced.
D. Parsons referred by friends.

5. A “full understanding” in research should be understood by the nurse


researcher as___.
A. Ensuring that participants are not placed at risk
B. Explaining the study including risks and ben
C. The right to decide voluntarily
D. Not exploiting information shared by participants

Situation – management of records is very vital in any health care facility.


The nurse must ensure there is due diligence in the task.

6. Nurse gay is assigned in the medical unit. She is guided that in


documentation, she should use abbreviation that is ____.
A. Used automatically to save precious time.
B. Reduced to the minimum in all units.
C. Approved standard list by the hospital.
D. Not used at all because it can be misinterpreted.

7. One error in record-keeping is illegible handwriting. What is the


apprppriate action by the nurse in this situation?
A. Request the senior nurse to read the order for you.
B. Let the resident-on-duty in the nurses station interpret it.
C. Call the physician who made the order.
D. Report the lapse to the quality assurance committee.

8. When a nurse commits an error in the progress notes the best action she
should do is to _____.
A. Cross the error many times to ensure it could no longer be read and sign.
B. Delete the erroneous phrase or sentence using correction fluid and sign.
C. Put a line across the sentence, make the correction over it, and sign.
D. Erase whatever is in error using a rubber eraser and sign.

9. Which is not a correct statement regarding record-keeping?


A. Failure to do it could be evidence of professional misconduct
B. It is an optional task to be done when circumstances allow.
C. It is a tool in professional practice that helps provide quality care.
D. It is part of the professional duty of the nurse.

10. The qa nurse conducts a regular audit of the medical records. The primary
purpose of conducting audit in a health facility is to _____.
A. Identify errors made by health personnel.
B. Identify arears for improvement
C. Ensure that standards are met.
D. Promote risk management.

Situation – health education to bong and his family is set up prior to


surgery. A program of weight gain aim for a high protein and high calorie
diet. The nurse prepares the health education plan.

11. The nurse ensures, which of the following should be present and be
cooperative in the educational program?
A. Patient, student nurses and interns
B. Patient, family and significant others
C. Head nurse and family
D. New staff nurses and nursing aides of the unit

12. The nurse must include in the education plan, which of the following
components? (select all that apply.)

I. Objectives
Ii. Content and time allotment
Iii. Teaching and learning resources
Iv. Evaluation parameters
A. Ii, iii, and iv
B. I, iii, and iv
C. I, ii, iii and iv
D. I, ii, and iii

13. To have a simplified and more understandable implementation of the plan,


the nurse presents it with use of ____.
A. Printed content in cartolina
B. A lecturer
C. A co-worker
D. Colored pictures

14. Before the education plan of the staff nurse can be finalized and
implemented, it is best that it is reviewed by the _____.
A. Nurse supervisor
B. Medical director
C. Head nurse
D. Chief nurse

15. During the implementation process, the nurse should ensure a____ for a
better assimilation of the teaching.
A. Serious lecturer
B. Lecture to start at 11:00am
C. Conducive time and place
D. Neophyte as sharer

Situation - nursing student myra decides to do a qualitative phenomenological


study on how the stigma of aids affects the patients. She has previously
identified 6 participants 3 teenage boys and 3 teenage girls.

16. What is the best way for myra to collect data from these participants?
A. Focus group discussion
B. Survey, questionnaire
C. Individual interview
D. Observation

17. What kind of sampling method should she apply?


A. Network
B. Random
C. Stratified
D. Purposive

18. The statements of the findings of the study that will be formulated by
myra should be by_______.
A. Summarizing the sharings of the participants of both sexes
B. Identifying the answers of the males and the females
C. Describing answers of the males and females by percentages
D. Extracting meanings and themes from significant statements

19. Informed consent in this study will be obtained by myra from the______.
A. Six participants only
B. Parents only
C. Six participants and available relative
D. Six participants and their parents

20. What is not important for myra to do when listening to tape recordings?
A. Do the listening as soon as possible after the interview.
B. Note for the voice tone and voice inflection.
C. Listen when she feels the motivation for a more productive time.
D. Take notice of the pauses of the participants.

Situation - communication is very important in a nurse-patient interaction


relationship.

21. It is not enough for the nurse to listen, but she also has to validate
what she has heard. The importance of validation are the following, except
__________.

A. Perceptions influence the interpretation of a message

B. Most patients are cognitively impaired

C. Eye contact does not necessarily send the same message


D. Assists clarifying thoughts

22. To be more responsible, a nurse needs to understand the elements of the


communication process. When she initiates interpersonal communication, the
element involved is __________.

A. Referent

B. Sender

C. Message

D. Channel

23. Should the nurse encounter patients who are stresses due to their health
condition, the best way to communicate is through which one of the following?

A. Sympathizing

B. Empathizing

C. Sharing

D. Listening

24. The reasons for the nurse wishing to enhance his/her communication skills
is to be able to establish rapport, except __________.

A. Brings about change to promote well-being of patients

B. Decreased incidents of legal problems

C. Gets better evaluation rating of care and delivery

D. Generates threat between the nurse and the patient


25. When the nurse interacts with patients face-to-face such as in getting
information during the assessment phase of the nursing process, the
level of communication is which of the following?

A. Intrapersonal

B. Interpersonal

C. Public communication

D. Verbal

Situation – it is necessary that records are well-recorded and properly kept


if they are to serve the organization and the requesting public well.

26. Incident reports (irs) shall be collected for the day and due
investigation, the irs _____.

A. Must be completed and stored on the open cabinet

B. Can be stored on the table top for easy retrieval

C. Must be summarized monthly and stored in a secured cabinet

D. Classified by date so they can be easily accessibility

27. The patient record (charts) are collected every three nights from the
various departments. The night nurse is expected to do the following,
except, _____.

A. Ensure t he correct order of the chart

B. See to the completeness of the chart pages

C. Bind the charts as they are

D. Tape or repair torn pages


28. When patient record reach the medical records, the assigned staff will
_____.

A. Store the charts in their respective shelves

B. Separate the medico-legal chats

C. Check the completeness of the charting of the doctors and nurses

D. Bind the chart immediately

29. The charts are stored in the medical records or storage room for at least
_____years.

A. 3 – 5 years

B. 1 – 5 years

C. 5 – 10 years

D. 1 – 3 years

30. How many years are medico-legal charts stored?

A. 8 years

B. 10 years after the case is closed

C. 5 years

D. 10 years

Situation - health education on hiv-aids has been massive in the years prior,
yet patients and their relative still have a number of queries and
misconceptions about it. Lerma, a young mother of 34, has been recently
diagnosed of the disease.
31. Lerma is aware that there is mother-to-child transmission of hiv-1. She
becomes concerned and asks the nurse when it specifically happens. The nurse
answers that it can occur in the following circumstances, except _____.

A. During breastfeeding

B. During casual contact

C. At the time of delivery

D. In utero

32. Lerma would like to know how she could limit further exposure to more hiv
virus by using preventive measures. The nurse inform her that the use of a
highly effective male condom that can decrease the transmission of hiv is
_____.

A. Non-latex

B. Latex

C. Polyurethrane

D. Lambskin

33. Patient lerma has a cd4 lymphocyte count which is below 200 cells/cumm.
She then asks what it means. The nurse answers that _____.

A. The result still falls within normal limits

B. She is in stage 3 hiv-aids

C. It is slightly below normal but there is nothing to worry

D. It is worrisome result but immediate attention is not necessary


34. The nurse counsels lerma that the prevention of hiv infection that is
usually not realistic is which one of the following?

A. Hiv testing

B. Behavioral interventions to reduce risks

C. Total abstinence

D. Linkage to a treatment center

35. Nurses are at risk for hiv exposure. Post exposure prophylaxis (pep) does
not include one of the following?

A. Take 2 – 3 anti – retroviral drugs, as prescribed.

B. Drugs must be taken for 28 days

C. Drugs must be taken for at least a week

D. Take the medicine within 72 hours of exposure.

Situation – the quality department has received numerous complaints, some of


them on patient falls.

36. Nonah who is an 86-year-old patient is admitted for fever fell from the
bed despite the presence of a watcher. The head nurse was concerned
since a fall protocol has been formulated for some time now. To avoid similar
incident, which of the following is most effective?

A. Interview the patient

B. Interview the nurse-on-duty

C. Investigate everyone

D. Do a root cause analysis


37. The nurse informed the head nurse that the lock in the side rails does not
work properly at times and might have gotten loose in the night thus the
incident of fall. What is the appropriate thing for the head nurse to do in
this regards?

A. Penalized all the nurses for not having reported such defect.

B. Make a memo to the maintenance department to check involved bed and the
rest of the beds.

C. Punish the nurse-on-duty for not reporting such observation before the
incident.

D. Warn every one this is not going to be tolerated in the future.

38. The quality improvement officer – in – charge of the unit plans to


conduct a meeting with the staff. The most important thing to tackle is to
_____.

A. Inform them of their punishment

B. Review the protocol

C. Inform them their lapses

D. Scold those involved

39. What is the initial action that the nurse should do immediately after the
fall mitigate the situation?

A. Document the incident right away.

B. Phone the head nurse to report the incident

C. Wait for the head nurse’s advice.

D. Have a doctor assess the patient immediately.


40. What lesson will the nurse learn from this incident?

A. Tell the others to cover her up.

B. Safety first – report even trivial but relevant observation.

C. Deny any error or omission.

D. Never get caught in her omissions.

Situation – nilda, 58 years of age, was brought to the er because of numbness


on her left face and arm and a confused mental state. The er doctor made an
initial impression of ischemic stroke.

41. Nilda wonders how she developed the manifestation of ischemic stroke. The
nurse explains that there is _____.

A. Extravasation of blood into the brain.

B. Possible presence of a cerebral aneurysm.

C. Vascular occlusion to the brain

D. Inability to understand spoken language

42. The nurse knows that visual-perceptual disturbances can occur in stroke.
When patient nilda manifests hemianopsia, she has _____.

A. Inability to perform everyday movements and gestures.

B. Blindness in half of the visual field

C. Difficulty speaking

D. Inability to understand spoken language


43. The nurse knows that initial diagnostic test that is ordered for stroke is
_____.

A. Carotid ultrasound

B. 12-lead ecg

C. Magnetic resonance imaging (mri)

D. Ct scan

44. The er nurse anticipates that a thrombolytic agent would be ordered to


treat ischemic attacks. Knowing the action of thrombolytic, the nurse must
particularly be alert for which adverse reaction?

A. Formation of blood clots

B. Bleeding

C. Early onset of infection

D. Allergies

45. Patient nilda complains of shoulder pain. The nurse is aware that she is
prone to have adduction of the shoulder. A nursing intervention for this is to
_____.

A. Position distal joint higher than proximal joint

B. Place a pillow under the arm to keep the arm close to the chest

C. Position the fingers so that they are barely flexed

D. Place one pillow in the axilla to keep the arm away from the chest.
Situation – emmy, 22 years of age is a midwife in the ob ward is now
complaining of contact dermatitis from gloves.

46. Patients like emmy who have experienced delayed hypersensitivity to latex
frequently complains of _____.

A. Flushing, bronchospasm

B. Urticaria, laryngeal edema

C. Rhinitis, conjunctivitis, blisters

D. Papules, vesicles, pruritus

47. The nurse knows that the diagnosis of contact latex allergy is based on
history and _____.

A. Latex specific ige

B. Finding ige in serum

C. Skin patch test

D. Elisa

48. Latex allergy can be a type i ige-mediated immediate hypersensitivity to


plant proteins from the latex of rubber. It can manifest in its most sever
form as _____.

A. Pruritus, erythema and swelling

B. Asthma

C. Anaphylaxis

D. Blisters and other skin lesions


49. Type i ige-mediated immediate hypersensitivity reaction is promptly manged
with _____.

A. Theophylline

B. Epinephrine

C. Corticosteroid

D. Dephenhydramine

50. The best prevention management of contact latex allergy is _____.

A. Applying lotion before gloving

B. Avoidance of latex products

C. Avoiding rubberized goods

D. Resigning from the job

Situation - sins of omission and commission may be committed by the nurse the
course of her duty. She must therefore be extra careful.

51. The patient for breast biopsy is very anxious and seemed not to understand
her ordered surgery, radical mastectomy. What is the appropriate action by
the nurse?

A. Call the supervisor to explain the procedure.

B. Have the available resident explain the surgery further.

C. Request the doctor to give the patient more information.

D. Supply the information missed to be explained by the doctor


52. The patient refused his intramascular injection but the nurse proceeded to
administer it anyway. What con the nurse be accused of?

A. Moral distress

B. Trespass to person

C. Assault

D. Battery

53. The staff nurse was doing prescribed modified steam inhalation to a
pediatric patient which resulted to burns. What is not a relevant statement to
establish negligence?

A. There is a duty to care.

B. There is a breach in the standard.

C. The breach caused the harm.

D. It was a verbal order by the physician.

54. Staff nurse a told her co-workers that staff b is suffering from
gonorrhea. What can staff a sued for?

A. Defamation

B. Slander

C. Discrimination

D. Libel

55. The surgical patient, a jehovah’s witness, reiterated non-acceptance to a


blood transfusion. The nurse defied it and the patient was transfused
intraoperatively. What case can be filed by the patient?
A. Moral distress

B. Battery

C. Trespass to person

D. Assault

Situation – nurses must closely adhere to the ethical principles and rules,
and not only to the laws.

56. When the nurse ensures that patients have consented to all treatments and
procedure, she is true to which ethical principle?

A. Fidelity

B. Beneficence

C. Veracity

D. Autonomy

57. If the nurse will refuse to perform duties for which she is not qualified,
she is practising _____.

A. Veracity

B. Beneficence

C. Respect

D. Non-maleficence

58. The code of ethics stipulates that human life is inviolable. Which
statement correctly translates the principle to the situation of the filipino
nurse as a professional?
A. It is okay to participate in euthanasia provided there is a doctor’s
order.

B. Participation is permissible when patient, family, and doctor’s order.

C. After the patient, doctor and hospital administrator have agreed, nurses
may choose to participate

D. Nurses shall not participate in euthanasia.

59. When the physician insists that his cancer patient undergoes radiation, in
addition to chemotherapy which is contrary to the patient’s and
family’s wishes, the physician is exercising _____.

A. Veracity

B. Autonomy

C. Fidelity

D. Paternalism

60. An iranian was admitted to the hospital for kidney transplantation. He


claims his donor is a filipino relative as required by the law. However,
it was discovered later that his claim was not true. Should the doctor fail to
act accordingly to the wrong information, the nurse is obligated to refer the
case to, which of the following?

A. Medical director

B. Administrator

C. Chief executive officer

D. Ethics committee
Situation – the nurse conducts health education on common types of allergy
with the parents of allergic children and adults with hypersensitivity issues.

61. During the conduct of the health education class, which communication
skill involves active listening that is being used by nurse to gain an
understanding of the patient’s message?

A. Clarifying

B. Responding

C. Attending

D. Confronting

62. The nurse must be alert about barriers to communication during the health
education class so that she can ______.

A. Use them when communicating

B. Communicate much better

C. Use them to enhance interactions

D. Rationalize wrong styles of communication

63. The mother of the atopic dermatitis patient is very concerned about
scarring that will result from the child’s frequent scratching. How will the
nurse best communicate her reassurance?

A. Asking the mother what she prefers for the child to have scars or he be
unable to sleep?

B. Telling the mother matter of fact that scars will lighten as the child
grows older anyway.
C. Tapping the hand of the mother while explaining that scarring occurs only
when lesions get infected.

D. Reminding the mother that beauty is only secondary to comfort derived from
scartching.

64. Because of the presence of skin lesions, atopic dermatitis affects the
patient’s self-esteem and his willingness to interact with others. The nurse
communicates her nursing intervention by _____.

A. Instructing the patient to go back to the primary physician

B. Giving instructions and counseling on preventive measures and treatments.

C. Referring the patient to the dermatologist

D. Referring the patient to the psychologist

65. The adult patient who was receiving diphenhydramine (benadryl) verbalized
he was always sleepy and fears he would sleep on the job and will get
fired. The nurse will best calm the patient by assuring him that his physician
can readily change his medicine to _____.

A. Chlorpheniramine (acrifed)

B. Brompheniramine (dimetapp)

C. Loratadine (allerta)

D. Dimenhydrinate (dramamine)

Situation – susie, 5 years old, is diagnosed with nephrotic syndrome and is


manifesting massive proteinuria massive proteinuria resulting to decreased
albumin in the blood.
66. The nurse understands that the passage of protein in the urine is the
result of _____.

A. Inherited kidney disorder

B. Increased glormuerular permeability

C. Rise in the production of albumin

D. Intrinsic kidney disease

67. Upon clinical assessment, the nurse observes that the outstanding
manifestation of the patient is _____.

A. Weight gain

B. Obesity

C. Emaciation

D. Edema

68. The physical appearance of the urine of the patient with nephrotic
syndrome is commonly _____.

A. Cloudy

B. Clear

C. Whitish

D. Frothy

69. The patient with nephrotic syndrome is ordered corticosteroids. Who of the
following are not allowed in the patient’s

A. Parents with diabetes

B. Relatives with upper respiratory tract infection.


C. Visitors with mild asthma.

D. Personnel with allergy

70. Corticosteroids are one of the main therapies for nephrotic syndrome.
Which of the following common side effects should the nurse watch for?

A. Loss of appetite

B. Increase in body hair

C. Loss of weight

D. Lowering of blood pressure

Situation - patient reno, 53 years of age, is pale and complains of easy


fatigability. He has undergone complete blood count (cbc) where abnormal cells
were found. He was diagnosed with acute lymphocytic leukemia (all).

71. The nurse ensure that the isolation procedure appropriate for reno is
_____.

A. Standard

B. Airborne precaution

C. Strict

D. Protective

72. Which of the following diagnostic procedures will definitely establish the
diagnosis for patient reno?

A. White blood cell count

B. Complete blood count


C. Bone marrow biopsy

D. Hemoglobin and hematocrit counts

73. When the thrombocyte count falls below 20,000/cu mm, the nurse will expect
that the physician will order _____.

A. Complete bed rest

B. Strict aseptic technique

C. Platelet transfusion

D. “limit visitors”

74. On the basis of his leukocyte count, the nurse instructs the patient not
to do, which one of the following?

A. Limit number of staff entering the room.

B. Be in a private room with the door closed always.

C. Receive immunization with a live attenuated virus.

D. Use antimicrobial soap when bathing.

75. During the period of exacerbation, the patient’s hemoglobin is markedly


decreased. What instruction by nurse is appropriate?

A. Allow exercise as long as tolerated.

B. Let patient be dependent on self-care.

C. Perform only activities of daily living.

D. Serve pork and liver barbecue.


Situation – nurses must continually grow as a person and as a professional.

76. As a newly licensed nurse employed in a tertiary hospital, you are


required to attend continuing professional development (cpd) program. When the
training program is for the enhancement of the competencies of nurses employed
in the hospital, it is called _____.

A. Self-directed

B. In-service training program

C. Informal training program

D. Formal education program

77. The professional career development of a nurse can be achieved through


various ways, such as_____.

A. Participating in political rallies

B. Attending culinary courses

C. Attendance in socio-civic activities

D. Engaging in cpd programs

78. When nurses are projected in a television advertisement as sex symbols,


what appropriate action is expected from a concerned nurse?

A. Go to the street to manifest displeasure of the nurses’ portrayal.

B. Report to the concerned agency

C. Condemn the issues in the radio program.

D. Keep your silence, it is the television station’s prerogative.


79. To enhance the personality of the graduate nurse, she/he may attend which
of the following programs?

A. Gymnastics

B. Marathon training

C. Social graces and physical fitness

D. Scuba diving

80. The nurse’s application to canada has finally been approved and she was
advised to depart in three months. She is currently enrolled in the
graduate school. Moreover, her mother has just been discharged from the
hospital. Which of the following actions is best?

A. Inform family that the money spent in graduate school can be easily earned
in canada.

B. Inform the agency that she could go anytime as they wish.

C. Share with friends that this is her escape from her sad life with her
family.

D. Request the recruiter to give her more time to settle her personal
concerns.

Situation - patient narding is diagnosed with stroke and suffers from a


number of deficits as a result of injury to his brain. His
rehabilitation may be long, depending on the extent of brain injury.

81. Narding has been urinating on and off in bed which is possibly related to
a flaccid bladder and difficulty in communicating. The nurse becomes concerned
because he is showing signs and symptoms of pressure sores due to his
immobility and the frequently wet beddings. The nurse decides to refer the
patient to the _____.
A. Resident on duty

B. Attending physician

C. Infectious doctor specialist

D. Supervisor – in – charge

82. Narding becomes unduly silent and keeps to himself after his stroke left
him with left-sided weakness. The nurse observes that the patient seems
really sad and shares the information to the physician, who refers the
patient to a ______.

A. Psychiatrist

B. Neurologist

C. Psychologist

D. Physiotherapist

83. The patient shared his concern about being able to father a child after
the stroke, especially that he has one child, a girl. He would like to
have two boys but he doubts his present sexual capacity. Who can best
help him along this area?

A. Primary consultant

B. Nurse

C. Supervisor in charge

D. Urologist

84. The patient should be assisted back to normal mobility after his stroke
partially incapacitated him. The referral that will be done by the attending
physician must be to which professional?
A. Occupational therapist

B. Resident physician

C. Physical therapist

D. Primary consultant

85. Narding seems forgetful, inattentive, and not answering appropriately to


some questions posed. The nurse anticipates that the consultant will need to
ask the professional help of the _____ for cognitive improvement.

A. Psychologist

B. Psychiatrist

C. Family doctor

D. Neurologist

95. Nurses may be privy to very personal information of patients and should
make every effort to make it confidential, otherwise she can be charged of
_____.

A. Negligence

B. Malpractice

C. Invasion of privacy

D. Defamatioin

Situation – quality improvement must be embraced by every health institution


if ti aims for safe and quality care. The medical director of camiguin medical
center made random rounds for five consecutive days to all areas of the
hospital to assess its services.
96. Which of the following is not a characteristic of quality improvement that
the medical director is interested in?

A. The leader is the empowered

B. Problem-solving is by everyone

C. The employees are treated as customers

D. Reacts to correct or bad situation

97. A risk is any event that causes problems or benefits on the healthcare
institution. The medical director knows that potential risks must be
identified across the hospital in order to prevent the following, except
_____.

A. Financial loss

B. Incident reports

C. Accidents

D. Injuries

98. Based on patient survey, a number of complaints have been tracked. Which
of the following is not a preventive activity in the practice of the medical
director’s risk management?

A. Fostering good administration-personnel relations.

B. Providing safe environment every time.

C. Tracking patient complaints at irregular intervals.

D. Satisfying patient needs and desires.


99. The medical director, with the chief nurse and other officers, of the
hospital believe that patient relations is impotant in risk management
which is not the way to handle complaints?

A. Let the patient express himself

B. Be sure to rebut the patient point by point

C. Staff should not be defensive.

D. Listen to the patient’s cue carefully.

100. Nurses usually complain they have no personal life because of rating
shifts. The following are three major ways to create personal time, except
_____.

A. Delegate work to others

B. Fill every moment with tasks or chores

C. Eliminate tasks that add no value

D. Hire someone else to do the work


Nursing practice v: care of clients with physiologic and psychosocial alterations,
part c

Situation – nurses inform patients taking antipsychotic medications about the


types of side effects that may occur. She encourages patients to report
instead of discounting the medications. Following are related to patient
teachings.

1. When taken antianxiety drugs like benzodiazopines, which appropriate


health teaching should the nurse emphasize?
a. Antianxiety drugs can treat the underlying problem
b. Patient should not drink alcohol because it potentiates its effect
c. Patient can discontinue the drug abruptly even without orders.
d. Patient can still drive his car cause of delayed response time

2. When taking anticonvulsant drugs like lithium, which appropriate health


teaching should the nurse emphasize?
a. Time of last dose must be accurate so that blood level monitoring be
accurate
b. Patient can take drugs even without food intake
c. Patient will not experience polyuria and polydipsia
d. Patient will have constipation, thus he has to increase fluid intake

3. Which of the following does not signify extrapyramidal symptoms (eps)


haldol?
a. Acute dystonia c. Pseudo parkinsonism
b. Akathisia d. Increased libido
4. The patient often appears restless, anxious, agitated with rigid posture
and lack of spontaneous gestures. Which of the following describes this
patient with intense need to move about?
a. Withdrawal c. Dystonia
b. Dyskinesia d. Akathisia
5. When taking ssri (selective serotonin reuptake inhibitors), which
appropriate health teaching should the nurse emphasize?
a. Aged cheese maybe allowed
b. Patient should take the drug first thing in the morning
c. Peanuts are allowed
d. Tyramise free diet can lower blood pressure

Situation – remedios, a 65-year-old housewife has been diagnosed with


rheumatoid arthritis both hands and knees.

6. On a visit to the clinic, a patient reports the onset of early symptoms


of rheumatoid arthritis. What will be the nurse focused assessment
during patient interview?
a. Enlarged nodules
b. Early morning stiffness of the lower extremities
c. Limited motion of joints of upper extremities
d. Deformed joints of the hands
7. Patient remedios complains she could not do household chores and her
knees hurt whenever she walks. Which nursing diagnosis would be most
appropriate?
a. A self-care deficit related to increasing joint pain
b. Activity intolerance related to fatigue and joint pain
c. Disturbed body image related to fatigue and joint pain
d. Ineffective coping related to increased joint pain
8. For a patient in the acute phase of rheumatoid arthritis, which of the
following should the nurse identify as lowest priority in the plan of
care?
a. Preserving joint function c. Relieving pain
b. Preventing joint deformity d. Maintaining usual task
9. A patient with osteoarthritis develops coagulopathy secondary to
long-term nonsteroidal anti-inflammatory drug (nsaid) use. The
coagulopathy is most likely the result of _____.
a. Decreased platelet adhesiveness
b. Blocked prothrombin conversion
c. Impaired vitamin k synthesis
d. Factor viii destruction
10.A nurse is teaching a patient with osteoarthritis about lifestyle
changes. The nurse kows the patient understand the teaching when she
states that she will ____.
a. Abstain from alcohol c. Loss weight
b. Avoid exercise d. Restrict caffeine

Situation – eighteen year old chimeya and her father came to clinic for
possible depression. She has a number of fears of getting sick and dying from
covid-19. She eats less and sleeps restlessly. She has not taken a bath for a
week, always talks about her missing mother who died due to covid-19
infection.

11.Working with a depressed chimeyam the nurse should understand that


depression is most directly related to a person’s ______.
a. Remembering her childhood
b. Stage in life
c. Having experienced a sense of loss
d. Experiencing poor interpersonal relationships with others
12.Early identification and treatment are essential to prevent long term
depression. Preventive measures do not include ____.
a. Medication as a treatment alone
b. Providing a stable home life
c. Practicing open and honest communication
d. Facilitating a strong sense of self trust, resilience and self-esteem
13.Chimeya was admitted in the hospital for treatment of her depression.
Which antidepressant drug is commonly used?
a. Norframin c. Prozac
b. Elavil d. Tofranil
14.To prevent the recurrence of depression, how long should the patient
take the anti depressant drugs?
a. Six months to two years c. One year to three years
b. Two months to one year d. One to three months
15.Three days after the admission of chimeya, the nurse observes she has
taken a bath, worn a clean dress, and combed her hair. What is the
appropriate reaction of the nurse to the behavioral change in chimeya?
a. “something is different about you today. What is it?”
b. “oh. I’m so pleased that you finally put on a clean dress”
c. “i see that you have worn a clean dress and have combed your hair”
d. “that’s good. You have on a clean dress and have combed your
hair.”

Situation – nurses provide their patients information they need to give


informed consent, only if it is within their scope of nursing practice and
nursing knowledge. Consent is the patients acknowledgement and acceptance of
medical treatment.

16.In emergency situation when a patient is unable to give consent for life
saing treatment, what type of consent allows to assume appropriate
medical treatment?
a. Implied consent c. Express consent
b. Informed consent d. Involuntary consent
17.Which of the following are essential components of informed consent?
i. Explanation of the procedure and alternatives to the procedure
ii. Discussion of potential risks and benefits of the procedure
iii. Confirmation that the patient understands the risks, benefits, and
any alterations
a. I, iii c. Ii, iii
b. I, ii d. I, ii, iii
18.Treatment of a patient without consent can constitute ______, which is
defined as intentional and unwanting touching.
a. Battery c. Negligence
b. Slander d. Tort
19.Whose responsibility is it to obtain informed consent?
a. Nurse manager
b. Anesthesiologist
c. Physician
d. Midwife
20.Bioethical issue should be describe as _______________________.
a. The withholding of food and treatment at the request of the patient in a
written
advance directive given before a patient acquired permanent brain damage
from an
accident
b. The physician’s making all decisions of client management without getting
input
from the patient
c. After the patient gives permission, the physician’s disclosing all
information
to the family for this support in the management of the patient
d. A research project that included treating all regular employed personnel
and not
treating all casual employed to compare the outcomes of specific drug
therapy

Situation – rosie, a new staff nurse, was assigned in the psychiatric unit a
depressed
patient assigned to her fell from her bed. Her head nurse asked her to submit an
incident
report (ir).

21. The purpose why the head nurse asked nurse rosie to submit an ir is to _____.

a. Note patterns of incidence in the same unit

b. Place it in nurse rosie’s201 file

c. Document immediately the incident

d. Evaluate nurse rosie’s performance

22. In writing the ir, which of the following in not included?

a. Who was / were involved?

b. What daily medications are given to the patient

c. What happened?

d. Who witnessed the incident?

23. What guideline is important in relation to incident report (ir)? It is _____.

a. Not made part of the patient’s chart

b. Placed in the nurse’s 201 file


c. Filed in the nurses station

d. Filed in the records section of the hospital

24. Which of the following would prove that the nurse action carried out met the
standards
of care on falls?

a. Utilizing the nursing process in providing safe, quality nursing care

b. Documenting the procedures done

c. Carrying out the doctor’s order

d. Performing physical assessment

25. Should the investigation of the fall go further, which of the following is the
best
source of factual information?

a. Incident report

b. Nurse’s notes in the chart

c. Anecdotal record

d. Process recording

Situation – mrs. Del, 70-year-old retired teacher is diagnosed of dementia. She


lives
with her 24-year-old granddaughter. Nurse maxie attends to her when she goes for
her opd
checkups.

26. Mrs. Del must be aware that the most common chronic incidence that brings about
injury
among elderly persons is _____.

a. Rheumatic fever c. Gallbladder

b. Hip fracture d. Urinary tract infection

27. Nurse maxie should recognized that the most common psychogenic disorder among
elderly
persons is ______.
a. Depression c. Decreased appetite

b. Sleep disturbance d. Inability to concentrate

28. Which of the following is the most common cause of dementia among elderly
persons?

a. Parkinson’s disease

b. Alzhiemer’s disease

c. Amyotrophic lateral sclerosis

d. Multiple sclerosis

29. Which of the following symptoms is common to both the presenile and senile
dementia
associated with alzheimer’s disease?

a. Increased appetite

b. Loss of short-term memory

c. Inappropriate behavior

d. Inability to provide self-care

30. Patient with dementia suffers from “sundown syndrome”. Which nursing action
should
be included in this patient’s care plan?

a. Maintain consistent schedule and sequence of daily activities.

b. Integrate patient’s cultural preferences in to the care provided.

c. Serve warm beverage and snack in the early evening.

d. Provide opportunities for patient to learn and practice new skills.


Situation – nurse annie is assigned to sherry, a junior high school student, who is
treated for her bulimia.

31. Bulimia is best defined as a / an ______.

a. Disorder of the unknown origin associated with starving oneself

b. Pathological disorder of binging and vomiting

c. Phobic disorder of fear of obesity

d. Eating disorder associated with vomiting

32. What condition is not likely to develop in sherry?

a. Hyperkalemia c. Gastric ulcer

b. Tooth decay d. Rectal bleeding

33. Which of the following conditions may lead to death in a bulimic patient like
sherry?

a. Hypokalemia and cardiac arrhythmias and arrest

b. Metabolic acidosis and renal failure

c. Hyponatremia and circulatory collapse

d. Hypernatremia and congestive heart failure

34. Endocrine changes often result in a bulimic patient. Which of the following
would be
an expected change in sherry?

a. Delayed thyroid stimulating hormone response to hormone replacement therapy

b. Increased production of follicle stimulating hormone

c. Hypopituitarism

d. Decreased adrenocorticotropic hormone in response to cortisone


35. Amitriptyline, an anti-depressant is the drug of choice in treating bulimia.
What is a
common side effect of this drug?

a. Anticholinergic effects c. Urinary frequency

b. Cholinergic effects d. Diarrhea

Situation - mr. Rollan is diagnosed to have chronic schizophrenia.

36. To prevent relapses of schizophrenia with mr. Rollan , which of the following
nurse
anna should not encouraged mr. Rollan and his family?

a. Keep any troubling side effects of medications with nurses.

b. Practice stress reduction techniques.

c. To follow the medication regimen accurately.

d. Participate regularly in any other forms of treatment.

37. Choose one nursing strategy nurse anna should not use.

a. Speak in a low, calm tone of voice.

b. Let him interact with you while he is hallucinating.

c. Maintain a nonthreatening stance, keep a physical distance.

d. Maintaining safety for herself and mr. Rollan.

38. Choose on least anna’s nursing action while communicating with mr. Rollan.

a. “please let me know if i can be helpful.”

b. Check his use of ordered prn medication.

c. “i’ll let you sit here quietly and i will be at the nurse station.”
d. “i’m just checking in with you to see if there is anything you need right
now.”

39. Which of the following strategies would the nurse instruct the patient to do as
a
measure to prevent relapse?

a. Report changes in sleeping, eating and mood.

b. Block hallucinations during daily activities.

c. Take additional medications on days when mr. Rollan is “feeling bad”.

d. Take stress management class.

40. Which of the following identified ability of mr. Rollan that he can now
effectively
participate in rehabilitation?

a. Ability to concentrate. C. Ability to talk.

b. Ability to think. D. Ability to do listen.

Situation – mrs. Labrador, 75 years old, is in the clinic for the treatment of
acute
closed-angle glaucoma.

41. The physician would like to measure the intraocular pressure with non-contract
(air
puff) tonometer. While preparing patient for her examination, the nurse inform the
patient
that _____.

A. After the examination, a slight pain will experienced

B. Before the examination, a medication will be given

C. It is a painless procedure that has no side effects

D. During the ocular fundoscopy, atropine eye drop will be instilled

42. Which symptoms are associated with acute-closed angle glaucoma?

A. Diplopia and photophobia


B. Blurred vision and colored rings around lights

C. Episodic blindness and no pain

D. Sensation of curtain drawn across the visual field

43. The physician has prescirbed pilocarpine one percent eye drops every six hours.
The
expected outcome for this medication is to _____.

A. Dilate the pupil by paralyzing the ciliary muscle

B. Prevent dryness of the cornea and conjunctiva

C. Promote drainage of aqueous humor from anterior chamber

D. Reduce inflammation of the iris and choroid

44. The physician recommends peripheral iridectomy to relieve intraocular pressure.


He
prescribed meperidine hydrochloride (demerol) 50mg and atropine sulfate 0.3mg im as
the
preoperative medications. The nurse should _____.

A. Recognize that atropine sulfate is given preoperatively to dilate the pupil

B. Recognize this as a usual preoperative medication and administer it

C. Realize that the atropine sulfate is being given to dry up secretion

D. Notify the physician and question the order

45. Which of these nursing diagnoses should the nurse give priority for an elderly
patient
who has impaired vision due to glaucoma?

A. High risk for injury

B. Impaired physical mobility

C. Grooming self-care deficit

D. Feeding self-care deficit


Situation – rizal, a 50 – year old house painter, came in with the chief complaints
of
slurred speech, blurring of vision, and hallucinations. Upon evaluation of his
case, it
revealed volatile substance intoxication.

46. The predisposing factor in rizal’s case is his _____.

A. Age

B. Occupation

C. Home environment

D. Community

47. Volatile substance abuse is considered the most dangerous among the abused
psychoactive substances because of the risk of _____.

A. Violence

B. Developing schizophrenia

C. Irreversible damage to the bone marrow, brain, liver, kidney

D. Malnutrition

48. The nurse heard rizal saying, “my mother visited me last night and reminded me
to
take care of myself.” This is manifestation of _____.

A. Auditory hallucination

B. Visual hallucination

C. Delusion

D. Reaction formation

49. Substance abuse affects not only the user but also the other members of the
family.
Which of the following is the most appropriate nursing diagnosis in the care of
rizal?

A. Impaired social interactions.


B. Impaired parenting

C. Dysfunctional family processes

D. Ineffective coping

50. Sever intoxication to volatile substances may lead to unconsciousness or even


death.
The priority nursing intervention in caring for such patient is monitoring the
_____.

A. Mental status

B. Neurological functions

C. Nutritional status

D. Vital signs

Situation – darwin, 35-year-old engineer met a vehicular accident while going to


work. He
suffered head injury, responsive and admitted at the intensive care unit for
close
monitoring and management.

51. During nursing assessment, darwin speaks a rambling manner and is unable to
repeat
words spoken to him. Which are of the brain most likely is affected?

A. Wernicke’s area

B. Broca’s area

C. Foramen magnum

D. Brodmann’s area

52. The physician orders a computerized transverse axial tomogram (cat) scan.
Nursing
preparation of the patient for this procedure includes:

A. Explaining that the vital signs will be monitored for 2 hours after the
examination.

B. Reassuring that cat scanning is a noninvasive procedure.

C. Explaining that a spinal tap will be done so that a radioactive isotope can be
injected.
D. Telling patient that a radiopaque dye is injected into artery in the arm.

53. The physician order to observe for early signs of increased intracranial
pressure
which includes _____.

A. Restlessness and change in level of consciousness

B. Elevated temperature and decerebrate posturing

C. Rising blood pressure and bradycardia

D. Widening pulse pressure and dilated pupils

54. All of the following signs indicate increased intracranial pressure except?

A. Decreased level of consciousness

B. Tachycardia

C. Papilledema

D. Vomiting

55. The nurse noticed dressing is wet. Which action by the nurse can be safely used
to
determine if the drainage contains cerebrospinal fluid (csf)? What is the attending
nurse
should do?

A. Blot the drainage with sterile gauze pad and look for a clear wet ring around
the spot
of blood.

B. Swab the orifice of the ear with a sterile applicator and send the specimen to
the
laboratory.

C. Obtain a negative reading for sugar after testing csf with tes-tape.

D. Gently suction the ear and send the specimen to the laboratory.

Situation – nurse manager participates in quality improvement projects to increase


awareness and achieve better performance of nursing team.
56. A professional practice system that manages clinical care of patients across a
continuum using managed care concepts and tools is called _____.

A. Modular nursing

B. Differentiated practice

C. Case management

D. Primary nursing

57. What is the outcome of having a sound clinical care management by professional
healthcare team?

A. It decreases patients’ length of stay

B. It diminishes collegiality between health care providers

C. It increases cost of hospitalization

D. It contributes to duplication of services

58. During a staff meeting, the nurse manager presents his own analysis of problems
and
proposals for actions to the staff, inviting critique and comments. Which
answer
indicates the manager’s leadership style?

A. Laissez-faire

B. Autocratic

C. Participative leadership

D. Democratic

59. Which of the following is often associated with the concept of decentralized
decision
making in management?

A. Team nursing

B. Interdisciplinary practice model

C. Shared governance

D. Primary nursing
60. Some decisions are best made by a group rather than by the nurse alone. What is
an
advantages of group decision making?

A. Promote collective contribution of ideas

B. Different ideas and opinions

C. Individual opinions are influenced by others

D. Dependency is fostered

Situation - glory, a 23 year old evening cashier of seven eleven 24 hours


convenience
store, was sexually abused by a jeepney driver while on her way home from work one
evening. She was brought to the er with bruises all over body. She was crying
uncontrollably and appears to be anxious.

61. Which of the following therapeutic communication should nurse ann say for
glory?

A. “you are upset, calm yourself first glory. I can’t understand you.”

B. “can you identify your abuser?”

C. “i know something terrible and horrifying happened to you.”

D. “would you like to relate to me what happened?”

62. In providing nursing care for glory during her acute stress reaction to rape
trauma,
nurse ann may apply, which of the following?

A. Physical assessment

B. Collaborate with community agencies

C. Crisis intervention techniques

D. Teaching and learning principles


63. Glory’s physical assessment is complete and physical evidence has been
collected.
After three days, nurse ann noted glory to be withdrawn, confused and at
times
physically immobile. How should nurse ann interpret these behaviors?

A. Evidence that the client is a high suicide risk

B. Signs of depression

C. Indicative of the need for longer hospital admission

D. Normal reactions to a devastating event

64. Emergency care to be given for rape victims are as follows:

I. If victim calls the hospital, tell her not to take a bath, wash or change
clothes, just
go directly to the hospital.

Ii. Provide privacy and be-judgmental

Iii. Stay with the victim, focus on physical safety and emotional security

Iv. Assist on pelvic examination to collect evidence as semen stains

A. I, iii, iv c. Ii, iii, iv

B. I, ii, iii d. I, ii, iv

64. Nurse ann wanted to become a patient advocate to rape victims. Which of the
following responsibilities should she note?

A. Isolate the patient first to provide privacy while attending to other patients.

B. Call the press since this is a legal case.

C. Perform thorough physical assessment and document objectively all evidence of


rape.

D. Postpone the physical examination until the patient is calm.

Situation – head nurse alona ensures teamwork and collaboration in her until to
achieve
efficient shared decision-making and open communication to provide safe patient
care.
66. A nurse returns from vacation and finds a new model of i.v. Pump attached to
her
patient’s i.v. How should the nurse proceed?

A. Read the i.v. Pump manual before caring for the patient.

B. Refuse to care for the patients

C. Inform the charge nurse and ask her to provide a teaching session about how to
use the
pump.

D. Use the pump because it is somewhat like the old pumps on the unit.

67. A nurse is caring for a 72-year-old male patient who requires insertion of a
central
venous catheter. Who is responsible for obtaining informed consent?

A. Physician who will insert the catheter

B. Charge nurse

C. Attending physician

D. The nurse assisting with the procedure

68. A nurse reports that a patient coughs frequently after taking anything by
mouth. The
dietitian recommends a swallow evaluation for the patient in which the physician
participating in the team rounds writes the order. This is an example of
collaboration
of client care _____.

A. With the ancillary care providers

B. Between the physician and the dietary deparment

C. With the risk management team because of risk for aspiration

D. Among members of the multidisciplinary group

69. Before delegating to a new nurse the task of giving a shower to a paraplegic
elderly,
the charge nurse should first ensure that the new nurse _____.

A. Has demonstrated competency for the task

B. Has received the assignment during endorsement time


C. Is supervised at all times

D. Provides companion to the patients

70. Which of the following tasks would be appropriate for the nurse to delegate to
nursing aide?

A. Assist a new postoperative patient to the bathroom

B. Teach a patient how to administer discharge medications.

C. Change a central line dressing.

D. Assist the patient during meal time.

Situation – mr. Ferrer, 42 years old, is admitted to the hospital in a semi-


conscious
state diagnosed with cerebrovascular accident.

71. The nurse obtains history of patient’s present illness from his family. What
significant information can the nurse gather from the patient’s family?

A. Consistent hypertension and dizziness

B. Palpitations and hypotension

C. Family history about illness

D. Emotional response to past illness

72. The priority nursing care for mr. Ferrer during the acute phase is to _____.

A. Provide sensory stimulation

B. Maintain respiratory and cardiac function

C. Prevent contracture and deformities

D. Maintain optimal nutrition


73. Part of nursing care plan is to observe mr. Ferrer for signs of increased
intracranial
pressure. Which of the following clinical manifestations would indicate this
condition?

A. Tachycardia and drop in blood pressure

B. Bradycardia and rising blood pressure

C. Bradycardia and drop in blood pressure

D. Tachycardia and rising blood pressure

74. Which of the following positions will be most appropriate to mr. Ferrer’s care?

A. Head of bed elevated in a lateral position

B. Head of bed elevated in a supine position

C. Right lateral position

D. Left lateral position

75. Mr. Ferrer’s wife is very upset asks if there is any hope to recover from his
condition? Which of the following is the most appropriate reply by the nurse?

A. “you must be patient, let’s hope for the best outcome.”

B. “you should never lose hope”

C. “it is too soon to tell what the outcome will be.”

D. “actually, manifestations may even get worse.”

Situation – liela, 5 years old, was diagnosed as autistic since she was 1 year old.

76. What behavior will nurse raffy observe as characterized by liela?

A. Inappropriate behavior, poor attention span with impulsivity

B. Negativistic hostile and defiant behavior

C. Failure to develop interpersonal skills


D. Anxiety induced involuntary stereotype motor movements

77. At her age, liela is in what stage of social development?

A. Initiative vs guilt

B. Trust vs mistrust

C. Industry vs inferiority

D. Autonomy vs shame and doubt

78. Nurse raffy reconnizes which of the following as a common behavioral sign of
autism?

A. Clinging behavior towards parent

B. Early language development

C. Indifference to being hugged or held

D. Creative imaginative play with peers

79. The best nursing intervention that nurse raffy can use to provide trusting
relationship with an autistic liela is to _____.

A. Convey warmth through touch

B. Early language development

C. Indifference to being hugged or held

D. Creative imaginative play with peers

80. Which pharmacologic treatment is appropriate for liela’s temper tantrum,


aggressiveness, self injury and stereotyped behavior?

A. Clonidine (catapres) c. Clomipramine (anafranil)

B. Naltrexone (re via) d. Haloperidol (haldol)


Situation - patient safety remains a global health care challenge. There are basic
principles of infection control. These include standard precautions and
transmission
based precautions. The following are transmission based precaution question.

81. Which one of the following is considered the most important intervention in
infection
control?

a. Personal protective clothing

b. Prevention of infection associated with catheter

c. Safe use and disposal of sharps

d. Hand hygiene of healthcare staff

82. Which mode of infection transmission is due to splashes of blood / body fluids
into
the mucosa or contamination of non intact skin with infected blood and body
fluids?

a. Ingestion c. Inoculation

b. Airborne d. Direct / indirect contact

83. Which mode of infection transmission is due to microorganism being transferred


to
other patients from contaminated equipments and via the hands of nurses?

a. Ingestions c. Inoculation

b. Airborne d. Direct / indirect contact

84. What mode of transmission is due to contaminated food and water being consumed?

a. Inoculation c. Ingestion

b. Direct / indirect contact d. Airborne

85. Which of the following is not a standard precaution?

a. Respiratory hygiene c. Personnel protective


equipment
b. Injection safety d. Hand hygiene

Situation – the nurse abides with ethico moral principles.

86. When the nurse placed the patient in restraints before using other methods of
intervention, she/he violated the patient’s rights to _____.

a. Receive confidential and respectful care

b. Provide informed consent

c. Receive treatment in the least restrictive environment

d. Refuse treatment

87. Which of the following actions is a violation of a psychiatric patient’s


rights?

A. Paranoid patient with delusions about his family is told that if he makes a
will, it
might not be valid.

B. The nurse confiscated the cellphone from patient’s room and tell him it is being
locked in the vault.

C. Staff members confiscated written letters done by patients addressed to local


newspaper.

D. Patient is paid minimum wage for helping in the hospital kitchen.

88. Which of the following in not covered in patient’s bill of rights?

a. Refusal to treatment c. Right to treatment

b. Informed consent d. Civil commitment

89. A patient has been advised by the psychiatrist that he needs inpatient
hospitalization. The patient agrees, signs the admission forms, and agrees to
receive
treatment. What type of admission is this?

a. Formal c. Informal
b. Voluntary d. Involuntary

90. Disclosure of client information beyond the interdisciplinary team without


consent of
the client is a breach of _____.

a. Confidentiality c. Duty

b. Beneficence d. Veracity

Situation – nursing research is conducted to answer a question or resolve problems


on the
relevance of the nursing profession.

91. The nurse develops the following hypothesis: elderly women receive less
aggressive
treatment for terminally-ill spine patients than younger women. Which variable
would be
considered to be the independent variable?

a. Degree of treatment received

b. Age of the patients

c. Use of inpatient treatment

d. Type of complications being treated

92. The following are considered qualitative research process except.

a. Sample c. Hypothesis

b. Literature review d. Data collection

93. Which of the following is an example of a primary source in a research study?

A. A textbook of medical-surgical nursing

B. A doctoral dissertation that critiques all research in the area of attention


deficit
disorder

C. A published commentary on the findings of another study.


D. A journal article about a study that used large, previously unpublished
databases
generated.

94. What is the best source to use when conducting a level one (1) systematic
meta-analysis of the literature?

a. An electronic database and doctoral dissertations

b. An electronic database

c. Doctoral dissertations

d. The cochrane statistical methods

95. Which type of research allows researchers to be neutral observers?

a. Quantitative research c. Case studies

b. Ethnographic research d. Qualitative researcher

Situation – mrs. Gomez, 63-year-old admitted for cataract extraction. Nurse lucy is
assigned to prepare the patient for surgery.

96. Mrs. Gomez tells the nurse that she does not want to know about her surgery.
What
would be the best response of the nurse?

a. “i must go over certain information with you.”

b. “you are right; do not worry yourself tonight.”

c. “you really sound quite concerned about your surgery.”

d. “well, i could talk to your son about this instead.”

97. What should the nurse do before giving pre-operative teaching to the patient?

a. Determine mrs. Gomez anxieties, level of understanding and expectations.

b. Research the surgical procedure so as to give step-to-step explanation.


c. Schedule teaching to begin 2-3 hours before surgery.

d. Give mrs. Gomez general information because specifics might be treatening.

98. While doing health teaching to mrs. Gomez, the attending nurse can best
recognize that
her patient is learning by _____.

a. Demonstrating a positive change in her behavior

b. Constant verbal reaffirmations that she understands

c. Her ability to repeat what was discussed

d. Nonverbal acknowledgment that she understands, such as nodding.

99. The nurse prepare mrs. Gomez for discharge. What would be the nurse most
important
post cataract surgery instruction to her patient?

a. Avoid the use of laxative

b. Use an eye shield at night

c. Avoid to touch the eye dressing

d. Curtail most heavy activities

100. After discharge, mrs. Gomez attends the eye clinic for follow-up visit. When
she
received her cataract glasses, it is important that the nurse advise her that
_____.

a. Magnification by the lens is only about 10 percent

b. Daily eye drops are required with eyeglasses

c. Her peripheral vision will be increased

d. Objects will appear closer than they really are

You might also like